Naked Science Forum

Non Life Sciences => Physics, Astronomy & Cosmology => Topic started by: Alan McDougall on 31/05/2016 06:04:43

Title: If Energy is neither created nor used up, where did energy come from?
Post by: Alan McDougall on 31/05/2016 06:04:43
If Energy is neither created nor used up, where did energy come from in the first place?

Alan
Title: Re: If Energy is neither created nor used up, where did energy come from?
Post by: JohnDuffield on 31/05/2016 14:03:58
I don't know the answer to this.

LOL, if I did, the wife would be looking at hotels in Stockholm.
Title: Re: If Energy is neither created nor used up, where did energy come from?
Post by: Alan McDougall on 31/05/2016 15:08:06
I don't know the answer to this.

LOL, if I did, the wife would be looking at hotels in Stockholm.

Nevertheless , why not give it a bash?
Title: Re: If Energy is neither created nor used up, where did energy come from?
Post by: JohnDuffield on 31/05/2016 20:48:29
Because I've thought about it for years, and I can't come up with anything that offers any kind of explanation. I'm not fond of creation ex-nihilo, and I find the universe has always existed  unsatisfactory too. As to where space or energy came from and how the universe began, well, that's where I hit the buffers. I just don't know.
Title: Re: If Energy is neither created nor used up, where did energy come from?
Post by: RobC on 31/05/2016 21:31:00
This question is worst than "Do dark energy/dark matter really exist?

Surely it's in the category of "we will never find out".
Title: Re: If Energy is neither created nor used up, where did energy come from?
Post by: JohnDuffield on 31/05/2016 22:00:47
Dark matter and dark energy really exist.

Shucks, that's an easy one!
Title: Re: If Energy is neither created nor used up, where did energy come from?
Post by: PmbPhy on 31/05/2016 22:51:52
Quote from: Alan McDougall
If Energy is neither created nor used up, where did energy come from in the first place?
The answer to your question is very easy to understand. To begin to understand your question you need to have a firm grasp of what energy is. I strongly recommend reading The Feynman Lectures on Physics - Volume I by Feynman, Leighton, and Sands, Addison Wesley, (1963)(1989). Read Section 4-1 What is energy? which starts on page 4-1. You can download that text here:
http://book4you.org/book/2047593/a467ef

The website is free to use. Registration is free too but you must register. It's worth it because it's an awesome website. You can download practically any physics text that you'd like from that site. The main page is: http://book4you.org/

However, if you don't want to go through the entire business of registering for the site, downloading the book and reading the section then you can go to my website and read What is Energy?[/b ] at:
http://www.newenglandphysics.org/physics_world/cm/what_is_energy.htm

From that page you'll learn that energy is not a physical entity, i.e. it's not a substance of any kind. Energy is merely a bookkeeping system which keeps track of a set of quantities, the sum of which is a constant of motion. The value of that constant (i.e. the total energy) is not important since in nearly all cases you can change it and not alter the physics (Even in the exceptions you can probably argue that the constant can be set to zero. However, that's an entirely different discussion). The important thing to keep in mind is that there are energies which have a negative value which can cancel those values of energy which are positive.

So the energy never came from anywhere since it's not a physical substance. However there is a real meaning to the values of the energy. But its the total that you're interested and that total for the entire universe can be set to zero.

In The Inflationary Universe, Alan Guth explains why the total energy of the universe is zero. You can read that appendix online. I placed it on my website at:
http://www.newenglandphysics.org/ask_a_physicist/guth_grav_energy.pdf
Title: Re: If Energy is neither created nor used up, where did energy come from?
Post by: JohnDuffield on 31/05/2016 23:15:13
Pmb: I must beg to differ.

Matter is made of energy. This is what Einstein's E=mc² paper (https://www.fourmilab.ch/etexts/einstein/E_mc2/www/) is all about (Einstein wrote L instead of E). He made it clear that "the mass of a body is a measure of its energy-content". He also said "if a body gives off the energy L in the form of radiation, its mass diminishes by L/c²". And we now know about electron-positron annihilation. That's where two bodies give off all their energy, and then they don't exist any more. Radiation is a form of energy. Energy is not just some book-keeping abstraction.

Also note the Foundation of the General Theory of Relativity (http://einsteinpapers.press.princeton.edu/vol6-trans/197?highlightText=gravitatively%20) where Einstein says "the energy of the gravitational field shall act gravitatively in the same way as any other kind of energy". Gravitational field energy is positive, not negative. The total energy of the universe is not zero. If you let two bodies fall towards one another, conservation of energy applies. You do not end up with less energy than you started off with. Gravity converts potential energy into kinetic energy, and when you dissipate this you're left with a mass deficit. But the books always balance, and they don't add up to zero.   
Title: Re: If Energy is neither created nor used up, where did energy come from?
Post by: jeffreyH on 01/06/2016 08:09:30
John that is why you will never be any good at science. You don't understand the simplest of concepts. You are also another cut and paste merchant. Stop spreading disinformation, go away and learn the mathematics. I might then take you seriously. And I don't mean go to google and find some equations to paste here.
Title: Re: If Energy is neither created nor used up, where did energy come from?
Post by: Alan McDougall on 01/06/2016 15:06:38
From that page you'll learn that energy is not a physical entity, i.e. it's not a substance of any kind. Energy is merely a bookkeeping system which keeps track of a set of quantities, the sum of which is a constant of motion. The value of that constant (i.e. the total energy) is not important since in nearly all cases you can change it and not alter the physics (Even in the exceptions you can probably argue that the constant can be set to zero. However, that's an entirely different discussion). The important thing to keep in mind is that there are energies which have a negative value which can cancel those values of energy which are positive.

Nonsense!
Title: Re: If Energy is neither created nor used up, where did energy come from?
Post by: impyre on 01/06/2016 15:21:45
John that is why you will never be any good at science. You don't understand the simplest of concepts. You are also another cut and paste merchant. Stop spreading disinformation, go away and learn the mathematics. I might then take you seriously. And I don't mean go to google and find some equations to paste here.
That seems a little combative.
In regards to the OP, I'm certain you already know that no one can offer any evidence on this... so I'll assume you want opinions. Mine is that the energy was provided by an external universe in the creation of a black hole, and that the big bang was the formation of the singularity as viewed from the inside. A space where the laws of physics are starting to break down, so new ones are born. Maybe just a silly fantasy, but it's a cool one. :p
Title: Re: If Energy is neither created nor used up, where did energy come from?
Post by: PmbPhy on 02/06/2016 20:53:37
Quote from: Alan McDougall
Nonsense!
Spoken like someone who has no understanding of energy. What I just said is well-known by any well-educated physicist. It's not a personal opinion of mine. You didn't read Feynman's text on energy like I suggested, did you? If you did then you wouldn't have made such a rude remark such as this.

Try actually learning about energy before making anymore ignorant comments like that. E.g. read From Alchemy to Quarks by Sheldon L. Glashow (Won the Nobel Prize for his work on the electroweak interaction). Read Chapter 3 - Energy and Momentum page 104.
Quote
In Hindu myth, Vishnu appears as nine avatars: fish, turtle, pig, monster, dwarf, Krishna, Buddha, and Rama the creator or the destroyer. As a white-winged horse, he will one day destroy Earth. Vishnu has much in common with energy. It, too, is an abstract quantity that is difficult to define because it comes in many guises. It's central property (shared with momentum and angular momentum) is conservation - energy can be neither made nor lost. Our primary source of energy is the Sun, but in a far off time it will fulfill Vishnu's prophecy:
The Sun is destined to explode and engulf the Earth.
The key remarks here are that energy is an abstract quantity and that its difficult to define. In fact its thought that it actually cannot be properly defined.

More later.
Title: Re: If Energy is neither created nor used up, where did energy come from?
Post by: PmbPhy on 02/06/2016 22:10:33
The following is from chapter 4 of The Feynman Lectures - Volume I.
Quote
4-1 What is energy?

In this chapter, we begin our more detailed study of the different aspects of physics, having finished our description of things in general. To illustrate the ideas and the kind of reasoning that might be used in theoretical physics, we shall now examine one of the most basic laws of physics, the conservation of energy. There is a fact, or if you wish, a law, governing all natural phenomena that are known to date. There is no known exception to this law—it is exact so far as we know. The law is called the conservation of energy. It states that there is a certain quantity, which we call energy, that does not change in the manifold changes which nature undergoes. That is a most abstract idea, because it is a mathematical principle; it says that there is a numerical quantity which does not change when something happens. It is not a description of a mechanism, or anything concrete; it is just a strange fact that we can calculate some number and when we finish watching nature go through her tricks and calculate the number again, it is the same. (Something like the bishop on a red square, and after a number of moves—details unknown—it is still on some red square. It is a law of this nature.) Since it is an abstract idea, we shall illustrate the meaning of it by an analogy.

Imagine a child, perhaps “Dennis the Menace,” who has blocks which are absolutely indestructible, and cannot be divided into pieces. Each is the same as the other. Let us suppose that he has 28 blocks. His mother puts him with his 28 blocks into a room at the beginning of the day. At the end of the day, being curious, she counts the blocks very carefully, and discovers a phenomenal law—no matter what he does with the blocks, there are always 28 remaining! This continues for a number of days, until one day there are only 27 blocks, but a little investigating shows that there is one under the rug—she must look everywhere to be sure that the number of blocks has not changed. One day, 4-1 however, the number appears to change—there are only 26 blocks. Careful investigation indicates that the window was open, and upon looking outside, the other two blocks are found. Another day, careful count indicates that there are 30 blocks! This causes considerable consternation, until it is realized that Bruce
came to visit, bringing his blocks with him, and he left a few at Dennis’ house. After she has disposed of the extra blocks, she closes the window, does not let Bruce in, and then everything is going along all right, until one time she counts and finds only 25 blocks. However, there is a box in the room, a toy box, and the mother goes to open the toy box, but the boy says “No, do not open my toy box,” and screams. Mother is not allowed to open the toy box. Being extremely curious, and somewhat ingenious, she invents a scheme! She knows that a block
weighs three ounces, so she weighs the box at a time when she sees 28 blocks, and it weighs 16 ounces. The next time she wishes to check, she weighs the box again, subtracts sixteen ounces and divides by three. She discovers the following:

(number of blocks seen) + [ (weight of box) − 16 ounces]/3 ounces  =  constant.    (4.1)

There then appear to be some new deviations, but careful study indicates that the dirty water in the bathtub is changing its level. The child is throwing blocks into the water, and she cannot see them because it is so dirty, but she can find out how many blocks are in the water by adding another term to her formula. Since the original height of the water was 6 inches and each block raises the water a quarter of an inch, this new formula would be:

(number of blocks seen) +[ (weight of box) − 16 ounces]/3 ounces
+ [ (height of water) − 6 inches]/1/4 inch = constant.                                                            (4.2)

In the gradual increase in the complexity of her world, she finds a whole series of terms representing ways of calculating how many blocks are in places where she is not allowed to look. As a result, she finds a complex formula, a quantity which has to be computed, which always stays the same in her situation.

What is the analogy of this to the conservation of energy? The most remarkable aspect that must be abstracted from this picture is that there are no blocks. Take away the first terms in (4.1) and (4.2) and we find ourselves calculating more or less abstract things. The analogy has the following points. First, when we are calculating the energy, sometimes some of it leaves the system and goes away, or sometimes some comes in. In order to verify the conservation of energy, we must be careful that we have not put any in or taken any out. Second, the energy has a large number of different forms, and there is a formula for each one. These are: gravitational energy, kinetic energy, heat energy, elastic energy, electrical energy, chemical energy, radiant energy, nuclear energy, mass energy. If we total up the formulas for each of these contributions, it will not change except for energy going in and out.

It is important to realize that in physics today, we have no knowledge of what energy is. We do not have a picture that energy comes in little blobs of a definite amount. It is not that way. However, there are formulas for calculating some numerical quantity, and when we add it all together it gives “28”—always the same number. It is an abstract thing in that it does not tell us the mechanism or the reasons for the various formulas.
Again, as I keep saying, energy is an abstract thing just as Feynman agrees.
Title: Re: If Energy is neither created nor used up, where did energy come from?
Post by: Alan McDougall on 02/06/2016 22:32:21
Energy is just vibrating particles left over fro the big bang!

The primordial energy has being dissipated, by the relentless progress of increasing entropy,  into the universe which is cooling down as particles become much less active in the near absolute zero of the dying universe.

Heat is nothing more than particles vibrating from a source of the big bang.

When fundamental particles stop moving, then at absolute zero nothing will ever happen again

Alan
Title: Re: If Energy is neither created nor used up, where did energy come from?
Post by: jeffreyH on 02/06/2016 22:33:25
John that is why you will never be any good at science. You don't understand the simplest of concepts. You are also another cut and paste merchant. Stop spreading disinformation, go away and learn the mathematics. I might then take you seriously. And I don't mean go to google and find some equations to paste here.
That seems a little combative.
In regards to the OP, I'm certain you already know that no one can offer any evidence on this... so I'll assume you want opinions. Mine is that the energy was provided by an external universe in the creation of a black hole, and that the big bang was the formation of the singularity as viewed from the inside. A space where the laws of physics are starting to break down, so new ones are born. Maybe just a silly fantasy, but it's a cool one. :p

Well John says things that are just not true. Then he will accuse those pointing out his errors of gaining their knowledge via 'pop science magazines'. Of course without any shred of evidence to suggest where others learn about physics. Which is a pursuit that John avoids like the plague. Just ask him some probing questions and you'll soon get the idea.
Title: Re: If Energy is neither created nor used up, where did energy come from?
Post by: JohnDuffield on 02/06/2016 22:42:25
That seems a little combative.
In regards to the OP, I'm certain you already know that no one can offer any evidence on this...
It isn't a little combative, impyre. It's dishonest. See my post above where I referred to Einstein, twice. It was Einstein who said radiation is a form of energy and the mass of a body is a measure of its energy-content. Google on conversion of matter to energy (https://www.google.co.uk/#q=%22conversion+of+matter+to+energy%22). This is what E=mc² is all about. Energy is real, matter is made of it. You are made of it.
Title: Re: If Energy is neither created nor used up, where did energy come from?
Post by: PmbPhy on 02/06/2016 22:46:20
Quote from: Alan McDougall
Energy is just vibrating particles left over fro the big bang!
Which clearly shows that you have no idea what translational kinetic energy or potential energy is (neither of which alone is associated with vibration). A particle moving at constant speed, like most of the particles making up the interstellar gas in the universe, has kinetic energy, none of which is vibrating.

After all this time posting in this forum I find it amazing how little you know about the basics of physics.
Title: Re: If Energy is neither created nor used up, where did energy come from?
Post by: jeffreyH on 02/06/2016 22:58:34
That seems a little combative.
In regards to the OP, I'm certain you already know that no one can offer any evidence on this...
It isn't a little combative, impyre. It's dishonest. See my post above where I referred to Einstein, twice. It was Einstein who said radiation is a form of energy and the mass of a body is a measure of its energy-content. Google on conversion of matter to energy (https://www.google.co.uk/#q=%22conversion+of+matter+to+energy%22). This is what E=mc² is all about. Energy is real, matter is made of it. You are made of it.

So you can post links and mention famous physicists. Yet you can't even answer a simple question on the FLRW metric. You posted the link on it to support a point you were making. Do you understand it or not? It is a simple question. You can put this to bed right now. Show everybody that you are not just smoke and mirrors.
Title: Re: If Energy is neither created nor used up, where did energy come from?
Post by: Alan McDougall on 03/06/2016 04:57:59
Quote from: Alan McDougall
Energy is just vibrating particles left over fro the big bang!
Which clearly shows that you have no idea what translational kinetic energy or potential energy is (neither of which alone is associated with vibration). A particle moving at constant speed, like most of the particles making up the interstellar gas in the universe, has kinetic energy, none of which is vibrating.

After all this time posting in this forum I find it amazing how little you know about the basics of physics.

Energy is exactly how I described it, and my knowledge of physics is not "BASIC"  THE VIBRATIONS OF FUNDAMENTAL PARTICLES, THAT WERE SET IN MOTION AT THE MOMENT THAT OUR UNIVERSE EMERGED OUT OF THE BIG BANG SINGULARITY

As an analogy think of a string of a musical instrument, you pluck it giving it energy to vibrate, thus; you are the source of the energy that made the string vibrate and "You are a real material thing"

Now replace yourself with the big bang which is the source of all energy that exists in all its forms in the universe, it is all still in the universe but dissipated due to the increasing entropy. You no longer play a part in you have plucked the string and left it vibrating until the vibration stopped.

To then go further to find the true source of all energy we must find the prime mover using increasing regressing until we are left with God as pure energy, the real thing, because God is light (energy) and in him is no darkness whatsoever.

I know there is no way to capture an amount of hypothetical energy and put it into a container, just as in my example we cant put you in container and call you bottle of pure energy. You represent the Big Bang, plucking the string of the universe setting all things in motion!
Title: Re: If Energy is neither created nor used up, where did energy come from?
Post by: agyejy on 03/06/2016 05:42:41
https://profmattstrassler.com/articles-and-posts/particle-physics-basics/mass-energy-matter-etc/matter-and-energy-a-false-dichotomy/ <- Matter and energy are very different things. Matter is a thing and energy is a property of things. Thus things can be made of matter but they cannot be made of energy.

https://profmattstrassler.com/articles-and-posts/particle-physics-basics/mass-energy-matter-etc/mass-and-energy/ <- Mass is a property and energy is a property and there is a relation between mass, energy, and momentum but the aren't equivilent.
Title: Re: If Energy is neither created nor used up, where did energy come from?
Post by: Alan McDougall on 03/06/2016 22:25:56

Physicists define energy as the capacity of a physical system to do work. It exists in several forms such as heat, mechanical energy, light, electrical energy etc

According to the Law of Conservation of energy, the total  energy of a system remains constant, though energy may transfer into another form.

Einstein realised that matter is actually energy in another form and his famous formula E=mc2 has been demonstrated to hold true. In other words Energy equals mass times speed of light squared.
 
So, where did energy come from? This is actually asking the question :Where did our universe come from? I would say it came from the Creator God who planned it and brought it into being. Since we are told in Scriptures that God exists before the universe, we can conclude that he used some of his energy to create all the energy of our universe. How did he do that? We can only speculate but I like to suggest that

God exists in a Timeless Dimension that pulsates with energy of that Timeless Dimension. Our universe is locked into time and so I like to imagine that the Creator locked a very precise amount of this timeless energy into time, so creating the universe in a flash of immense energy.
 
Scientific discoveries are showing that for our universe to exist there had to be a large number of extremely finely-tuned factors, such as the relationship between the known forces of nature. If any one of these factors was only fractionally different we would not exist. Therefore, to my mind the Big Bang Creation is a wonderful description of the moment when God locked some of the Timeless Dimension energy into the limitations of time.

Personally I see no conflict between the Big Bang and God.
 

(Give a better candidate for God as the source of all energy?)
Title: Re: If Energy is neither created nor used up, where did energy come from?
Post by: Colin2B on 05/06/2016 07:10:19
https://profmattstrassler.com/articles-and-posts/particle-physics-basics/mass-energy-matter-etc/matter-and-energy-a-false-dichotomy/ <- Matter and energy are very different things. Matter is a thing and energy is a property of things. Thus things can be made of matter but they cannot be made of energy.

https://profmattstrassler.com/articles-and-posts/particle-physics-basics/mass-energy-matter-etc/mass-and-energy/ <- Mass is a property and energy is a property and there is a relation between mass, energy, and momentum but the aren't equivilent.
Alan,
These are really worth reading. I'm a great fan of Matt Strassler, he doesn't use the pop science terminology that seems to dog physics reporting in the popular science press.
Also worth reading are these articles on PmbPhy's web site http://www.newenglandphysics.org/other/other.htm
Title: Re: If Energy is neither created nor used up, where did energy come from?
Post by: Alan McDougall on 05/06/2016 13:49:02
https://profmattstrassler.com/articles-and-posts/particle-physics-basics/mass-energy-matter-etc/matter-and-energy-a-false-dichotomy/ <- Matter and energy are very different things. Matter is a thing and energy is a property of things. Thus things can be made of matter but they cannot be made of energy.

https://profmattstrassler.com/articles-and-posts/particle-physics-basics/mass-energy-matter-etc/mass-and-energy/ <- Mass is a property and energy is a property and there is a relation between mass, energy, and momentum but the aren't equivilent.
Alan,
These are really worth reading. I'm a great fan of Matt Strassler, he doesn't use the pop science terminology that seems to dog physics reporting in the popular science press.
Also worth reading are these articles on PmbPhy's web site http://www.newenglandphysics.org/other/other.htm

By the way I was a member of Pmbphy's site but he did not like anyone challenging him and dominated it like a dictator.

As for the link, thank you, I think everyone in this thread should read the article, not just me although I admit I am an engineer not a physicist, but a person interested in learning as much as possible as advanced physics at this late stage of my rather protracted life span of almost 76 years young.

Yes compared to some of you, my understanding of the deep maths in physics is basic, however when compared to the average Joe on the high street it is highly advanced.

What we do not need is to be mocked or accused of dishonesty when our take on a subject is wrong.

Alan
Title: Re: If Energy is neither created nor used up, where did energy come from?
Post by: JohnDuffield on 05/06/2016 17:07:01
...I'm a great fan of Matt Strassler, he doesn't use the pop science terminology that seems to dog physics reporting in the popular science press...
Unfortunately Matt Strassler's article is at odds with what Einstein said and what E=mc² is all about: radiation is a form of energy, and matter is made of energy. IMHO you can confirm the latter for yourself by considering Compton scattering (http://hyperphysics.phy-astr.gsu.edu/hbase/quantum/comptint.html):

(https://www.thenakedscientists.com/forum/proxy.php?request=http%3A%2F%2Fi.stack.imgur.com%2Fw9OB2.gif&hash=73eab1cc41b5bffc10ece7371157f8db)Image courtesy of Rod Nave's hyperphysics

When you perform Compton scattering, some of the photon's E=hc/λ wave energy is converted into electron kinetic energy. If you repeat the process and perform another Compton scatter using the scattered photon, then another and another and another, in the limit you remove all of the photon wave energy, whereupon there's no wave left. The photon has then been entirely converted into electron kinetic energy. This is why light can be viewed as *just* kinetic energy, or why light is a "form of energy".  The important thing to note is that in pair production you can convert the photon into an electron and a positron, so you can say the electron is quite literally made from kinetic energy. You made matter out of energy. The electron is made out of the same thing that causes electrons to move. Then when you annihilate the electron it with a positron you get two photons, which are just kinetic energy, and you're back where you're started. 

NB: I would add that IMHO it's better to speak of energy-momentum rather than energy alone. You can think of energy as a distance-based measure of energy-momentum, and momentum as a time-based measure of energy-momentum. They're two sides of the same coin in that you can't reduce the kinetic energy of the cannonball in space without reducing its momentum. 
Title: Re: If Energy is neither created nor used up, where did energy come from?
Post by: PmbPhy on 05/06/2016 21:20:29
Quote from: Colin2B
These are really worth reading. I'm a great fan of Matt Strassler, he doesn't use the pop science terminology that seems to dog physics reporting in the popular science press.
Also worth reading are these articles on PmbPhy's web site http://www.newenglandphysics.org/other/other.htm
Although Dr. Strassler and I don't agree on everything I do admire him. His website is really good, i.e. very informative and well-written. Thanks for the kudos regarding my website. Did you read the following webpage:
http://www.newenglandphysics.org/physics_world/cm/what_is_energy.htm

Feynman and myself share the same concept of energy. I quoted him in reply #12. Did you read it or my webpage above? There are many journal articles about this subject in the physics literature. If anybody wants to read any of those articles all they have to do is go to: http://booksc.org/ and type What is energy? into the search window, click "search" and then a list of articles will appear. Do a search using the phrase "What is energy" and you'll see that 8 items will be listed.

Now Alan and John have a list of physics journal articles to read on the subject of greatest importance in this thread. Do you think that either of them will take our advice and read one of them? :)
Title: Re: If Energy is neither created nor used up, where did energy come from?
Post by: jeffreyH on 05/06/2016 21:31:26
...I'm a great fan of Matt Strassler, he doesn't use the pop science terminology that seems to dog physics reporting in the popular science press...
Unfortunately Matt Strassler's article is at odds with what Einstein said and what E=mc² is all about: radiation is a form of energy, and matter is made of energy. IMHO you can confirm the latter for yourself by considering Compton scattering (http://hyperphysics.phy-astr.gsu.edu/hbase/quantum/comptint.html):

(https://www.thenakedscientists.com/forum/proxy.php?request=http%3A%2F%2Fi.stack.imgur.com%2Fw9OB2.gif&hash=73eab1cc41b5bffc10ece7371157f8db)Image courtesy of Rod Nave's hyperphysics

When you perform Compton scattering, some of the photon's E=hc/λ wave energy is converted into electron kinetic energy. If you repeat the process and perform another Compton scatter using the scattered photon, then another and another and another, in the limit you remove all of the photon wave energy, whereupon there's no wave left. The photon has then been entirely converted into electron kinetic energy. This is why light can be viewed as *just* kinetic energy, or why light is a "form of energy".  The important thing to note is that in pair production you can convert the photon into an electron and a positron, so you can say the electron is quite literally made from kinetic energy. You made matter out of energy. The electron is made out of the same thing that causes electrons to move. Then when you annihilate the electron it with a positron you get two photons, which are just kinetic energy, and you're back where you're started. 

NB: I would add that IMHO it's better to speak of energy-momentum rather than energy alone. You can think of energy as a distance-based measure of energy-momentum, and momentum as a time-based measure of energy-momentum. They're two sides of the same coin in that you can't reduce the kinetic energy of the cannonball in space without reducing its momentum.

So John can you enlighten us to the meaning of theta and phi in Compton scattering?
Title: Re: If Energy is neither created nor used up, where did energy come from?
Post by: jeffreyH on 05/06/2016 23:19:29
Matter and mass are not equivalent. Matter does not increase relativistically. Mass is not a given for any particle without interaction with other fields.The mechanism of relativistic mass increase is unknown. We can say it is velocity related but no more. It is not a subject that you can make glib statements about.
Title: Re: If Energy is neither created nor used up, where did energy come from?
Post by: PmbPhy on 06/06/2016 00:41:14
Quote from: jeffreyH
The mechanism of relativistic mass increase is unknown. We can say it is velocity related but no more. It is not a subject that you can make glib statements about.
Why on Earth would you say that my friend? W sure do know the mechanism for relativistic mass increase. Just follow the derivation at: http://www.newenglandphysics.org/physics_world/sr/inertial_mass.htm

Once you go through it and fully understand what you're reading then you'll understand why mass increases with speed. It's because when the speed increases the time and distance measurements change and those changes manifest themselves by changes in momentum and the combination of momentum and speed define mass.

By the way. I've sent you a PM and an e-mail and you didn't respond to either of them. Is there a reason why?
Title: Re: If Energy is neither created nor used up, where did energy come from?
Post by: JohnDuffield on 06/06/2016 08:29:39
Now Alan and John have a list of physics journal articles to read on the subject of greatest importance in this thread. Do you think that either of them will take our advice and read one of them?
I've read them all, along with Einstein's E=mc² paper (https://www.fourmilab.ch/etexts/einstein/E_mc2/www/). That's where Einstein referred to radiation as a form of energy, and where we learned that matter is made of energy. 

You can't counter my Compton scattering explanation that supports this, can you?   
Title: Re: If Energy is neither created nor used up, where did energy come from?
Post by: puppypower on 06/06/2016 11:47:39
Energy is the bridge between the speed of light ground state of the universe, and all the inertial states of the universe. Photons travel at the speed of light which is the same in all inertial references. This leg of the energy bridge is connected to the C ground state. Photons will also red and blue shift relative to inertial references. This leg of the bridge is connected to inertial reference.

If we traveled at the speed of light the inertial universe would appear as a point-instant. This means at the speed of light, one cannot see the variety of energy we see in inertial reference. At the speed of light ground state, one can only see one wavelength; infinite wavelength, where photons approach zero energy. The reason is at the speed of light, shorter wavelengths will contract to a fraction of a point-instant, which is not mathematically possible. The variety of wavelengths of energy we see connected to the inertial side of the bridge.

Current theory does not use a speed of light ground state, therefore energy remains more nebulous. But with a C ground state, since mass cannot move at the speed of light; special relativity, we need an intermediate state or a bridge, with one leg in both places.

Title: Re: If Energy is neither created nor used up, where did energy come from?
Post by: jeffreyH on 06/06/2016 18:07:22
Now Alan and John have a list of physics journal articles to read on the subject of greatest importance in this thread. Do you think that either of them will take our advice and read one of them?
I've read them all, along with Einstein's E=mc² paper (https://www.fourmilab.ch/etexts/einstein/E_mc2/www/). That's where Einstein referred to radiation as a form of energy, and where we learned that matter is made of energy. 

You can't counter my Compton scattering explanation that supports this, can you?

What is it about "you are on Pete's ignore list" that you don't understand?
Title: Re: If Energy is neither created nor used up, where did energy come from?
Post by: jeffreyH on 06/06/2016 18:10:08
John you specifically use Compton scattering to support your view so why not answer the trivial question posed about theta and phi. Is it because it destroys your assertion or because you simply don't know the answer.
Title: Re: If Energy is neither created nor used up, where did energy come from?
Post by: PmbPhy on 07/06/2016 07:41:31
Quote from: jeffreyH
What is it about "you are on Pete's ignore list" that you don't understand?
It's just his way of attempting to make me look ignorant.
Title: Re: If Energy is neither created nor used up, where did energy come from?
Post by: Alan McDougall on 07/06/2016 09:39:59
Quote from: Alan McDougall
Energy is just vibrating particles left over fro the big bang!
Which clearly shows that you have no idea what translational kinetic energy or potential energy is (neither of which alone is associated with vibration). A particle moving at constant speed, like most of the particles making up the interstellar gas in the universe, has kinetic energy, none of which is vibrating.

After all this time posting in this forum I find it amazing how little you know about the basics of physics.

Nothing amazing, from your lofty tower it seems you have reached a point of omniscience, leaving the rest of us all gasping, at your level of understanding of how everything in existence really works.

The kinetic theory of matter (particle theory) says that all matter consists of many, very small particles which are constantly moving or in a continual state of motion. The degree to which the particles move is determined by the amount of energy they have and their relationship to other particles. The particles might be atoms, molecules or ions. Use of the general term 'particle' means the precise nature of the particles does not have to be specified

What is wrong with that my friend?

Alan
Title: Re: If Energy is neither created nor used up, where did energy come from?
Post by: Colin2B on 07/06/2016 23:27:43
Feynman and myself share the same concept of energy. I quoted him in reply #12. Did you read it or my webpage above?
Excuse short reply, but low on time today, just back from trip.
I read Feynman lectures way back but the piece on energy didn't stand out because it confirmed the way I have always been taught to look at it - energy is not a 'thing' but a way of consistently counting a group of different but related properties.
The article on NEP I read last yr and is a useful reminder of the importance of considering the whole system. The example with ball and spring reminds me of the problem many people have in talking about energy. If we drop a steel ball we understand that it gains KE, and we are unlikely to confuse the ball as a thing and the energy as a property - no one would say the ball is energy. The same can be said for waves - they transfer energy but we would not call them pure energy. With less tangible things I have noticed people become confused and I have heard eg nuclear explosions and light described as pure energy - loose thinking.
The useful thing about the Strassler articles is that he covers quite succinctly the relationship between matter, mass, energy and momentum and use of momentum/energy in the calculations of interactions. He does take a hard line on relativistic mass (which I think is a useful concept) but I can see that in particle physics it's important to use a common methodology for interactions.
Unfortunately Matt Strassler's article is at odds with what Einstein said and what E=mc² is all about: radiation is a form of energy, and matter is made of energy. ......
When you perform Compton scattering, some of the photon's E=hc/λ wave energy is converted into electron kinetic energy. If you repeat the process and perform another Compton scatter using the scattered photon, then another and another and another, in the limit you remove all of the photon wave energy, whereupon there's no wave left. The photon has then been entirely converted into electron kinetic energy. This is why light can be viewed as *just* kinetic energy,.....
John
I'm surprised by your comments here. Current mainstream particle physicists refer to the energy of light as a property of light (the energy transferred by light) rather than describing light as energy eg http://sciencequestionswithsurprisinganswers.org/mobile/2015/01/12/why-is-light-pure-energy/.
Your 'proof' can also be applied to sound waves. A sound wave, which carries sound energy, will reflect in turn from multiple surfaces, transferring energy to the molecules in each reflector in the form of momentum which is dissipated as heat. Eventually the energy is used up and the wave disappears, however, we don't refer to sound, seismic, or other waves as "just energy".

I am also surprised by your comments regarding Strassler and Einstein and I don't see how they are at odds. In his article Strassler says:
"Einstein knew that energy and momentum were conserved according to previous experiments, so he sought (and found) equations that would preserve this feature of the world.  And he also discovered along the way that the mass of a system would have to satisfy equation E2 = (pc)2 + (mc2)2"           
Strassler then uses this equation as the starting point for going on to describe how conservation of energy and momentum are used in calculating interactions in particle physics.
If you feel there is a problem with his methodology it would help our understanding if you were to show us how you would perform the calculations in his examples and indicate where you feel he is in error.



Title: Re: If Energy is neither created nor used up, where did energy come from?
Post by: Colin2B on 07/06/2016 23:31:32
What is wrong with that my friend?
Alan, jeffferyH says PmbPhy won't read your posts, hence won't respond, because you and JD are on his ignore list
Title: Re: If Energy is neither created nor used up, where did energy come from?
Post by: jeffreyH on 08/06/2016 01:06:26
Feynman and myself share the same concept of energy. I quoted him in reply #12. Did you read it or my webpage above?
Excuse short reply, but low on time today, just back from trip.
I read Feynman lectures way back but the piece on energy didn't stand out because it confirmed the way I have always been taught to look at it - energy is not a 'thing' but a way of consistently counting a group of different but related properties.
The article on NEP I read last yr and is a useful reminder of the importance of considering the whole system. The example with ball and spring reminds me of the problem many people have in talking about energy. If we drop a steel ball we understand that it gains KE, and we are unlikely to confuse the ball as a thing and the energy as a property - no one would say the ball is energy. The same can be said for waves - they transfer energy but we would not call them pure energy. With less tangible things I have noticed people become confused and I have heard eg nuclear explosions and light described as pure energy - loose thinking.
The useful thing about the Strassler articles is that he covers quite succinctly the relationship between matter, mass, energy and momentum and use of momentum/energy in the calculations of interactions. He does take a hard line on relativistic mass (which I think is a useful concept) but I can see that in particle physics it's important to use a common methodology for interactions.
Unfortunately Matt Strassler's article is at odds with what Einstein said and what E=mc² is all about: radiation is a form of energy, and matter is made of energy. ......
When you perform Compton scattering, some of the photon's E=hc/λ wave energy is converted into electron kinetic energy. If you repeat the process and perform another Compton scatter using the scattered photon, then another and another and another, in the limit you remove all of the photon wave energy, whereupon there's no wave left. The photon has then been entirely converted into electron kinetic energy. This is why light can be viewed as *just* kinetic energy,.....
John

If you feel there is a problem with his methodology it would help our understanding if you were to show us how you would perform the calculations in his examples and indicate where you feel he is in error.

You may be waiting a while for the equations.
Title: Re: If Energy is neither created nor used up, where did energy come from?
Post by: Alan McDougall on 08/06/2016 03:47:14
What is wrong with that my friend?
Alan, jeffferyH says PmbPhy won't read your posts, hence won't respond, because you and JD are on his ignore list

No problem he does not like to be challenged on anything, that is why he created his own-forum (Now Defunct) and invited me to join because up until on our Naked Science forum he thought I was one of his admirers which I was not and am still not! 
Title: Re: If Energy is neither created nor used up, where did energy come from?
Post by: PmbPhy on 08/06/2016 04:20:44
What is wrong with that my friend?
Alan, jeffferyH says PmbPhy won't read your posts, hence won't respond, because you and JD are on his ignore list
Jeff said that because in the thread Does the universe have an edge
http://www.thenakedscientists.com/forum/index.php?topic=66960.msg488727#msg488727
I wrote in reply #39
Quote
Once again, you have absolutely no idea what you're talking about. The nature of the universe is based on general relativity and the Cosmological Principle (an axiom based on observations of the distribution of galaxies in the visible universe). That's why I keep saying that this is the way is based on theory and not mere speculation. From that and the measured density of matter in the universe the shape of the universe is determined. There's no speculation involved.

Since you keep making unfounded accusations and thus appear to have no real interest in learning what theory predicts and keep claiming that predictions are all speculation I can no longer justify either reading or responding to your comments. Welcome to my ignore list. You'll be in the company of the likes of John Duffield et al.
So why does Alan and JD keep asking me questions when they know they're on my ignore list? Perhaps Alan thinks I'm unable to respond to a challenge like JD claim where he wrote
Quote
You can't counter my Compton scattering explanation that supports this, can you?
That claim and any other like it is pure nonsense. There has never been an explanation that I assert is wrong that I can't prove and no serious challenge that I can't meet. But I will no longer attempt to explain things to people who can't understand it because they refuse to learn. E.g. I made an attempt to explain what energy is to Alan but all he could do is reply "Nonsense" showing me that he's not willing to learn. I gave up on those two for good.
Title: Re: If Energy is neither created nor used up, where did energy come from?
Post by: Colin2B on 08/06/2016 15:09:54
MODERATOR REQUEST

We appreciate that some of you have 'history' due to contact in other fora and that this will influence your conversations here. We allow a degree of lively discussion but this topic is becoming more personal than science. Please keep your replies on topic.
Thank you

PS - there are a number of members here who are valued for their knowledge of physics and the consistent, high quality of their replies. Some will be intolerant of incorrect or inaccurate science and may seem rather robust in their replies. We would ask both sides to cut the other a little slack, but any poster should not take a lack of response to indicate agreement with their post or that a response is not possible, everyone has the right to ignore.
Title: Re: If Energy is neither created nor used up, where did energy come from?
Post by: Alan McDougall on 08/06/2016 16:49:24
MODERATOR REQUEST

We appreciate that some of you have 'history' due to contact in other fora and that this will influence your conversations here. We allow a degree of lively discussion but this topic is becoming more personal than science. Please keep your replies on topic.
Thank you

PS - there are a number of members here who are valued for their knowledge of physics and the consistent, high quality of their replies. Some will be intolerant of incorrect or inaccurate science and may seem rather robust in their replies. We would ask both sides to cut the other a little slack, but any poster should not take a lack of response to indicate agreement with their post or that a response is not possible, everyone has the right to ignore.

It costs nothing to be polite, which will open the mind of the most stubborn learner!
Title: Re: If Energy is neither created nor used up, where did energy come from?
Post by: Alan McDougall on 10/06/2016 22:18:43
All the matter in the universe could been created from a bit of "primordial energy" or the big bang singularity 

This scenario is a consequence of applying Einstein's theory of gravity to the inflationary universe model. (Alan Guth) 

Thus the known laws of nature can in principle explain where the matter and energy in the universe came from, provided there was at least a seed of energy to begin with.

Exactly, what that seed of energy was admittedly baffles me, but maybe, somewhere, some-when and somehow, at this null alpha point; a the drummer struck the drum of existence, or a " Loaded primordial Spring" was sprung dissipating its force in an infinite moment" setting off a cascade of potential energy to activate the universe by the processes of increasing entropy?

Alan
Title: Re: If Energy is neither created nor used up, where did energy come from?
Post by: JohnDuffield on 12/06/2016 21:27:15
John
I'm surprised by your comments here.
Sorry Colin, I missed your post.

Current mainstream particle physicists refer to the energy of light as a property of light (the energy transferred by light) rather than describing light as energy eg http://sciencequestionswithsurprisinganswers.org/mobile/2015/01/12/why-is-light-pure-energy/.
Particle physicists describe light as having energy whilst Einstein described light as a form of energy. I'm with Einstein on this. As I said previously I take this view because of what we can work out from  Compton scattering (http://hyperphysics.phy-astr.gsu.edu/hbase/quantum/comptint.html):

(https://www.thenakedscientists.com/forum/proxy.php?request=http%3A%2F%2Fi.stack.imgur.com%2Fw9OB2.gif&hash=73eab1cc41b5bffc10ece7371157f8db)Image courtesy of Rod Nave's hyperphysics

When you perform Compton scattering, some of the photon's E=hc/λ wave energy is converted into electron kinetic energy. If you repeat the process and perform another Compton scatter using the scattered photon, then another and another and another, in the limit you remove all of the photon wave energy, whereupon there's no wave left. The photon has then been entirely converted into electron kinetic energy. This is why light is "just" kinetic energy, or why light is a "form of energy".  The important thing to note is that in pair production you can convert the photon into an electron and a positron, so you can say the electron is quite literally made from kinetic energy. You made matter out of energy. The electron is made out of the very same thing that makes electrons to move.   
 
Your 'proof' can also be applied to sound waves. A sound wave, which carries sound energy, will reflect in turn from multiple surfaces, transferring energy to the molecules in each reflector in the form of momentum which is dissipated as heat. Eventually the energy is used up and the wave disappears, however, we don't refer to sound, seismic, or other waves as "just energy".
The difference is that sound waves involve the motion of molecules, and we describe this energy as an attribute or property of those molecules. For light waves, there are no such molecules. 

I am also surprised by your comments regarding Strassler and Einstein and I don't see how they are at odds. In his article Strassler says:
"Einstein knew that energy and momentum were conserved according to previous experiments, so he sought (and found) equations that would preserve this feature of the world.  And he also discovered along the way that the mass of a system would have to satisfy equation E2 = (pc)2 + (mc2)2".       
Strassler then uses this equation as the starting point for going on to describe how conservation of energy and momentum are used in calculating interactions in particle physics. If you feel there is a problem with his methodology it would help our understanding if you were to show us how you would perform the calculations in his examples and indicate where you feel he is in error.
There's nothing wrong with E2 = (pc)2 + (mc2)2. What's wrong with Matt Strassler's article is this: "But energy is not itself stuff; it is something that all stuff has". That flatly contradicts Einstein. When he says "The stuff of the universe is all made from fields" he's giving a Standard Model viewpoint that's at odds with general relativity, which the Standard Model doesn't cover. In general relativity a gravitational field is space that's "neither homogeneous nor isotropic" (http://einsteinpapers.press.princeton.edu/vol7-trans/192?highlightText=%22neither%20homogeneous%22). See Einstein's 1929 article (http://www.rain.org/~karpeles/einsteindis.html) where he described a field as a state of space. General Relativity is mainstream, as is E=mc² along with the mass of a body is a measure of its energy-content. A body doesn't have energy like it has speed, it contains it. 
Title: Re: If Energy is neither created nor used up, where did energy come from?
Post by: jeffreyH on 13/06/2016 00:24:30
I am no longer ignoring John. So here goes. John I would very much appreciate your take on the standard model and Gell-Mann's eightfold way. Also on the predictive power of this model. I am sure you are fully aware of its history since you use it to support your view that the standard model is at odds with relativity. Take your time. No rush.
Title: Re: If Energy is neither created nor used up, where did energy come from?
Post by: Alan McDougall on 13/06/2016 01:48:24
Every object in the universe has potential energy! In my opinion the source of all the potential energy was the Big Bang, which was crudely put, was pure distilled,  inactive infinite energy, in a primordial container.

Somehow this non-created infinite pool of condensed, "as of yet inactive "stuff", was poured out into our universe from the singularity, to become active energy, in all its forms, that now sustains the processes of time and entropy enabling the universe to evolve and become the complex reality we all now exist in.

Reel back the above and all the energy reverts into the original infinite pool of condensed stuff that will morph into all forms of energy and start the while process again, without the loss of a single iota of potential energy. 
Title: Re: If Energy is neither created nor used up, where did energy come from?
Post by: jeffreyH on 13/06/2016 02:09:52
I am no longer ignoring John. So here goes. John I would very much appreciate your take on the standard mode and Gell-Mann's eightfold way. Also on the predictive power of this model. I am sure you are fully aware of its history since you use it to support your view that the standard model is at odds with relativity. Take your time. No rush.

You are so kind I weep!

Well John, who has no qualifications in physics at all, is trying to say that a physicist with the relevant qualifications is wrong. So he had better be able to back up that claim by showing he has the knowledge and understanding to support his assertions. Not just being good at finding things to cut and paste from google searches. Otherwise he is attempting to slur someone with an established reputation. If you think that is fine then stand up and be counted.
Title: Re: If Energy is neither created nor used up, where did energy come from?
Post by: Alan McDougall on 13/06/2016 03:15:54
I am no longer ignoring John. So here goes. John I would very much appreciate your take on the standard mode and Gell-Mann's eightfold way. Also on the predictive power of this model. I am sure you are fully aware of its history since you use it to support your view that the standard model is at odds with relativity. Take your time. No rush.

You are so kind I weep!

Well John, who has no qualifications in physics at all, is trying to say that a physicist with the relevant qualifications is wrong. So he had better be able to back up that claim by showing he has the knowledge and understanding to support his assertions. Not just being good at finding things to cut and paste from google searches. Otherwise he is attempting to slur someone with an established reputation. If you think that is fine then stand up and be counted.

My apologies!
Title: Re: If Energy is neither created nor used up, where did energy come from?
Post by: JohnDuffield on 13/06/2016 08:18:51
I am no longer ignoring John. So here goes. John I would very much appreciate your take on the standard model and Gell-Mann's eightfold way. Also on the predictive power of this model. I am sure you are fully aware of its history since you use it to support your view that the standard model is at odds with relativity. Take your time. No rush.
Start a thread and I'll tell you what I can.

Well John, who has no qualifications in physics at all, is trying to say that a physicist with the relevant qualifications is wrong. So he had better be able to back up that claim by showing he has the knowledge and understanding to support his assertions. Not just being good at finding things to cut and paste from google searches. Otherwise he is attempting to slur someone with an established reputation. If you think that is fine then stand up and be counted.
I've already backed up what I said by referring to Einstein. If you're saying the mass of a body is a measure of its energy-content is wrong, if you're saying radiation is a form of energy is wrong, if you're saying E=mc² is wrong, then it's you attempting to slur someone with an established reputation.
Title: Re: If Energy is neither created nor used up, where did energy come from?
Post by: jeffreyH on 13/06/2016 21:55:23
I am no longer ignoring John. So here goes. John I would very much appreciate your take on the standard model and Gell-Mann's eightfold way. Also on the predictive power of this model. I am sure you are fully aware of its history since you use it to support your view that the standard model is at odds with relativity. Take your time. No rush.
Start a thread and I'll tell you what I can.

Well John, who has no qualifications in physics at all, is trying to say that a physicist with the relevant qualifications is wrong. So he had better be able to back up that claim by showing he has the knowledge and understanding to support his assertions. Not just being good at finding things to cut and paste from google searches. Otherwise he is attempting to slur someone with an established reputation. If you think that is fine then stand up and be counted.
I've already backed up what I said by referring to Einstein. If you're saying the mass of a body is a measure of its energy-content is wrong, if you're saying radiation is a form of energy is wrong, if you're saying E=mc² is wrong, then it's you attempting to slur someone with an established reputation.

You assume to know what Einstein meant. You think that your one opinion outweighs the multitude of professionals working directly with the particles whose energy to presume to know all about. The subtleties of science elude you John. You are like the proverbial bull shopping for china.
Title: Re: If Energy is neither created nor used up, where did energy come from?
Post by: Alan McDougall on 13/06/2016 22:17:03
Einstein’s theory of general relativity, proposed that there were regions of space where a form of positive energy (or maybe anti-gravity?) was actually pushing space outward. As space expands, it releases stored up gravitational potential energy, which converts into the intrinsic energy that fills the newly created void.

Thus does the expansion of the universe violate the law of energy conservation and could it be the source from which all energy emerged to fill it with potential energy?

Molecules in motion=??
Title: Re: If Energy is neither created nor used up, where did energy come from?
Post by: jeffreyH on 13/06/2016 22:59:09
Energy is not a physical thing but an attribute of mass. Mass itself is not straightforward. John uses the term mass without specifying its type. Is it rest mass, inertial mass, gravitational mass or relativistic mass? These distinctions are important and are the exact type of subtleties that John show by his own words not to understand. It is too easy to take on board misconceptions and to believe that they are accepted science. It is a minefield for the layman. If in doubt question what you read and ask for other opinions. The best answers will come from moderators.
Title: Re: If Energy is neither created nor used up, where did energy come from?
Post by: Alan McDougall on 13/06/2016 23:19:32
Energy is not a physical thing but an attribute of mass. Mass itself is not straightforward. John uses the term mass without specifying its type. Is it rest mass, inertial mass, gravitational mass or relativistic mass? These distinctions are important and are the exact type of subtleties that John show by his own words not to understand. It is too easy to take on board misconceptions and to believe that they are accepted science. It is a minefield for the layman. If in doubt question what you read and ask for other opinions. The best answers will come from moderators.

Always from moderators?

My thoughts

Cause effect means that there something asymmetric has happened in the remote past. This early state of asymmetry is closely associated with the idea of information/energy transfer,  with the resulting enigma of information/energy happening, when the net displacement of space and time became a reality.

This restriction on information transfer is the same as those on energy transfer, and that the movement caused by the interaction of matter and antimatter, suddenly resulted in the movement of fundamental particles, effected by the primordial conflict between the two forms of opposing matter.

The very early universe was asymmetrical with equal amounts of matter and antimatter, and when they met they annihilated almost all of each other leaving mostly, say colossal gamma rays clouds, that over vast periods of time have dissipated and now all that remains in our universe might be a mere .0000000000001% of those original sources. Which had spewed out their contents, into our matter dominated universe.   

This primordial state of unbalance resulted in all atoms and molecules moving and by extrapolation became known to us as energy or its potential. All this energy that has been left over is now contained within the confines of our closed universe.

From just a very rough estimate, maybe only .000000000001% remain as usable energy from the original sources, that had previous held/contained within them all of two primordial forms of matter. (from this primordial source when they met and annihilated each other) 

Information, energy and time and space must be considered in the same picture.

The law of conservation of energy, also known as the first law of thermodynamics, states that the energy of a closed system must remain constant, it can neither increase nor decrease without interference from outside.

"My question then do we know for sure that the universe itself is a closed system"?, if not information could be leaking into it from the "Outside"?

The Big Bang might have been a "White Hole" ? or a leak from the outside?
Title: Re: If Energy is neither created nor used up, where did energy come from?
Post by: Colin2B on 14/06/2016 09:15:57

Always from moderators?
No, there are members whose contributions are judged to be reliable and who admit when they are mistaken eg PmbPhy, Ethos, JeffreyH. In this thread I would also mention agyegy who's contribution is sound.
Title: Re: If Energy is neither created nor used up, where did energy come from?
Post by: Alan McDougall on 14/06/2016 11:39:39

Always from moderators?
No, there are members whose contributions are judged to be reliable and who admit when they are mistaken eg PmbPhy, Ethos, JeffreyH. In this thread I would also mention agyegy who's contribution is sound.

Perhaps then I should considerably up my game, make my posts much more complex and profound, by the inclusion of much more detail and equations into my posts?  (I never seem to get any credit for my present method of posting). I have deliberately kept them as precise, simple and easy to read as possible, to enable the least informed member/visitor of having a real chance of actually understanding the answers to a particular topic question.

Maybe then I could join your list of the exalted few?

Alan
Title: Re: If Energy is neither created nor used up, where did energy come from?
Post by: Colin2B on 14/06/2016 14:30:33
I have deliberately kept them as precise, simple and easy to read as possible, to enable the least informed member/visitor of having a real chance of actually understanding the answers to a particular topic question.
That is the best way. More complex answers are only necessary in rare cases.
Title: Re: If Energy is neither created nor used up, where did energy come from?
Post by: jeffreyH on 14/06/2016 20:26:27

Always from moderators?
No, there are members whose contributions are judged to be reliable and who admit when they are mistaken eg PmbPhy, Ethos, JeffreyH. In this thread I would also mention agyegy who's contribution is sound.

Perhaps then I should considerably up my game, make my posts much more complex and profound, by the inclusion of much more detail and equations into my posts?  (I never seem to get any credit for my present method of posting). I have deliberately kept them as precise, simple and easy to read as possible, to enable the least informed member/visitor of having a real chance of actually understanding the answers to a particular topic question.

Maybe then I could join your list of the exalted few?

Alan

Not everyone reads every thread. Not every person that reads a thread will post a response. I don't see posting as some kind of competition. If I post something and get no replies then I move on and try to research the answers I want from other sources. People will be reading what you write and it may well be giving them something to think about. Listening is even more important than writing your own ideas down. I have learned an awful lot by doing just that.
Title: Re: If Energy is neither created nor used up, where did energy come from?
Post by: Alan McDougall on 14/06/2016 20:40:07

Always from moderators?
No, there are members whose contributions are judged to be reliable and who admit when they are mistaken eg PmbPhy, Ethos, JeffreyH. In this thread I would also mention agyegy who's contribution is sound.

Perhaps then I should considerably up my game, make my posts much more complex and profound, by the inclusion of much more detail and equations into my posts?  (I never seem to get any credit for my present method of posting). I have deliberately kept them as precise, simple and easy to read as possible, to enable the least informed member/visitor of having a real chance of actually understanding the answers to a particular topic question.

Maybe then I could join your list of the exalted few?

Alan

Not everyone reads every thread. Not every person that reads a thread will post a response. I don't see posting as some kind of competition. If I post something and get no replies then I move on and try to research the answers I want from other sources. People will be reading what you write and it may well be giving them something to think about. Listening is even more important than writing your own ideas down. I have learned an awful lot by doing just that.

I fully accept that, thank you!

I do carefully read the posts of the more informed members of the forum and have leaned a lot in the process, which is hopefully evident in the increasing quality of my own posts?

Alan

Alan
Title: Re: If Energy is neither created nor used up, where did energy come from?
Post by: jeffreyH on 14/06/2016 20:47:51
Just keep at it. Science is fascinating and a very extensive subject. Too much for anyone to learn properly. At the moment I am going to try to start reading again. I had stopped for a while but it will be reading on mathematics rather than physics.
Title: Re: If Energy is neither created nor used up, where did energy come from?
Post by: IAMREALITY on 14/06/2016 20:50:59
I could probably come up with dozens of theories, but they'd still have no more legitimacy than anyone else's, when it comes down to it.  Cause if it's one thing I've learned from my exploration of the universe and its wonders, is that literally ANYTHING is possible.  Anything at all.  Any theory could hold water, no matter how unreasonable or unlikely it sounds to another. 

So having that said, I guess I'll go in this direction.  We are taught energy can be neither created nor destroyed.  However we forget one small caveat: the fact that only applies to our universe, only to our set of physics, only to our 'reality' and within our own bounds.  No law of physics though, none at all, are said to be multiversally multiversal, if that makes sense (is that the first such a phrase has been uttered?  If so, I hereby lay claim to it!!!).  So we have no idea what the rules for energy or the creation/destruction of it would be external of our own universe, and therefore there can be a whole other set of theories as to where the energy that formed the singularity that has expanded ever since and condensed into all forms of matter may have come from.  Just something to ponder...
Title: Re: If Energy is neither created nor used up, where did energy come from?
Post by: IAMREALITY on 14/06/2016 20:57:52

Energy is not a physical thing but an attribute of mass. Mass itself is not straightforward. John uses the term mass without specifying its type. Is it rest mass, inertial mass, gravitational mass or relativistic mass? These distinctions are important and are the exact type of subtleties that John show by his own words not to understand. It is too easy to take on board misconceptions and to believe that they are accepted science. It is a minefield for the layman. If in doubt question what you read and ask for other opinions. The best answers will come from moderators.


Not sure I agree with this, energy being an 'attribute' of mass, and not a 'physical' thing, if by physical you mean actually existing as its own entity.  Spin, in relation to the spin of a particle, is an attribute.  Spin cannot turn into anything else, cannot take any other forms, it does not exist in any physical sort of sense.  The same cannot be said for energy; however, since all matter that exists condensed from a singularity of unimaginable energy.  And no, I'm not a physicist and am definitely a layman, so of course I attest that I may really be missing something in your comment.  But I'm just looking at it at face value and logically, and replying from that angle.

Title: Re: If Energy is neither created nor used up, where did energy come from?
Post by: Alan McDougall on 14/06/2016 21:03:20
I could probably come up with dozens of theories, but they'd still have no more legitimacy than anyone else's, when it comes down to it.  Cause if it's one thing I've learned from my exploration of the universe and its wonders, is that literally ANYTHING is possible.  Anything at all.  Any theory could hold water, no matter how unreasonable or unlikely it sounds to another. 

So having that said, I guess I'll go in this direction.  We are taught energy can be neither created nor destroyed.  However we forget one small caveat: the fact that only applies to our universe, only to our set of physics, only to our 'reality' and within our own bounds.  No law of physics though, none at all, are said to be multiversally multiversal, if that makes sense (is that the first such a phrase has been uttered?  If so, I hereby lay claim to it!!!).  So we have no idea what the rules for energy or the creation/destruction of it would be external of our own universe, and therefore there can be a whole other set of theories as to where the energy that formed the singularity that has expanded ever since and condensed into all forms of matter may have come from.  Just something to ponder...

You are speculating , there is absolutely no evidence of a macro universe, it has not even reached the stage of being considered a theory. Theories are based on some good evidence and there is of now, no evidence of macro or any other different universe, operating under different rules.

And even if there were other universes,  I simply cannot fathom how the conservation of energy would not apply in them as a fundamental law of physics?
Title: Re: If Energy is neither created nor used up, where did energy come from?
Post by: IAMREALITY on 14/06/2016 21:14:58
I could probably come up with dozens of theories, but they'd still have no more legitimacy than anyone else's, when it comes down to it.  Cause if it's one thing I've learned from my exploration of the universe and its wonders, is that literally ANYTHING is possible.  Anything at all.  Any theory could hold water, no matter how unreasonable or unlikely it sounds to another. 

So having that said, I guess I'll go in this direction.  We are taught energy can be neither created nor destroyed.  However we forget one small caveat: the fact that only applies to our universe, only to our set of physics, only to our 'reality' and within our own bounds.  No law of physics though, none at all, are said to be multiversally multiversal, if that makes sense (is that the first such a phrase has been uttered?  If so, I hereby lay claim to it!!!).  So we have no idea what the rules for energy or the creation/destruction of it would be external of our own universe, and therefore there can be a whole other set of theories as to where the energy that formed the singularity that has expanded ever since and condensed into all forms of matter may have come from.  Just something to ponder...

You are speculating , there is absolutely no evidence of a macro universe, it has not even reached the stage of being considered a theory. Theories are based on some good evidence and there is of now, no evidence of macro or any other different universe, operating under different rules.

And even if there were other universes,  I simply cannot fathom how the conservation of energy would not apply in them as a fundamental law of physics?

Ummmm, every single one of us are speculating son, that's the whole point of the exercise.    And though I'm not a fan of the multiverse myself, I am also forced to admit that there are plenty of reasons to accept that it actually is plausible.  There is nothing that has ruled it out and it is no longer considered a fringe idea. In fact, some of the best and brightest in the field accept the possibility.

Science since it's very first days has been limited by those with limited minds; who only can think as deep as their own narrow beliefs.  I choose to not be a slave to such limitations; even when dealing in angles I'm not a fan of.  I will still never allow my mind, nor my thought experiments, to have any boundaries. 

And it's not surprising to me that you are unable to fathom such a concept as you state, because you are in fact a slave of your self imposed mental limitations.  And it is silly to say there is no evidence of there being different rules in different universes, since it is likely that it would be impossible, no matter how advanced we get, to ever have any proof of anything beyond the bounds of our own universe.  So it will always be speculation based on the best theories or concepts our intelligent minds can indeed fathom.  And right now we're just only getting started.  But yes, there are not only reasons to believe the multiverse concept can be real, but also reasons to believe that physics may indeed act differently there.  Unless, of course, you believe Stephen Hawking to be a chump...

Title: Re: If Energy is neither created nor used up, where did energy come from?
Post by: Alan McDougall on 14/06/2016 22:09:57
IAMREALITY?

For me to be your son you would have to be at least 105 years old so please no patronising here keep it serious.

I see nothing wrong with speculation, however, there is another sub-forum for this type of debate

Maybe you should start a thread under 'New Theories" below? 

Thanks

Alan
Title: Re: If Energy is neither created nor used up, where did energy come from?
Post by: IAMREALITY on 14/06/2016 22:26:15
IAMREALITY?

For me to be your son you would have to be at least 105 years old so please no patronising here keep it serious.

I see nothing wrong with speculation, however, there is another sub-forum for this type of debate

Maybe you should start a thread under 'New Theories" below? 

Thanks

Alan

Sorry son, but I feel no need to start a new thread in a sub forum, when it is perfectly appropriate as an answer written here.  You asked a question.  I merely gave my opinion on it.  It's ya know, kinda like the point here and stuff. 

But you have yet to refute one thing I've said nor give any credible reason as to why my replies are any more invalid than anyone else's.  You seem to want to think you're the be all end all, but you've clearly shown to be the opposite.  You want to dabble in the deeper mysteries of the universe, the things we do not yet know, yet you start with boundaries and mental limitations that are not in any way conducive to discovery or the process.   You don't seem to like there to be any discussion about the multiverse for example, when it is actually a product of our physics and starting to be regarded more and more as a likelihood; albeit one that may always be impossible to prove.  But the things we can prove, and the theories we do have, and as a product of our own physics, the concept and even likelihood of a multiverse is very, very real...

So you're just gonna have to find a way to get over it I think.  I'm sure in time you will.

For now though, I will continue to reply to whatever thread I like, whenever I like, however I like, and do not require your permission nor your advisement with any of it.  For I will not ever take advice from a limited mind.  Thanks.
Title: Re: If Energy is neither created nor used up, where did energy come from?
Post by: Alan McDougall on 15/06/2016 03:37:35
IAMREALITY?

For me to be your son you would have to be at least 105 years old so please no patronising here keep it serious.

I see nothing wrong with speculation, however, there is another sub-forum for this type of debate

Maybe you should start a thread under 'New Theories" below? 

Thanks

Alan

Sorry son, but I feel no need to start a new thread in a sub forum, when it is perfectly appropriate as an answer written here.  You asked a question.  I merely gave my opinion on it.  It's ya know, kinda like the point here and stuff. 

But you have yet to refute one thing I've said nor give any credible reason as to why my replies are any more invalid than anyone else's.  Fact is, you've got nuttin pal.  You seem to want to think you're the be all end all, but you've clearly shown to be the opposite.  You want to dabble in the deeper mysteries of the universe, the things we do not yet know, yet you start with boundaries and mental limitations that are not in any way conducive to discovery or the process.   You don't seem to like there to be any discussion about the multiverse for example, when it is actually a product of our physics and starting to be regarded more and more as a likelihood; albeit one that may always be impossible to prove.  But the things we can prove, and the theories we do have, and as a product of our own physics, the concept and even likelihood of a multiverse is very, very real...

So you're just gonna have to find a way to get over it.  I'm sure in time you will.

For now though, I will continue to reply to whatever thread I like, whenever I like, however I like, and do not require your permission nor your advisement with any of it.  For I will not ever take advice from a limited mind.  Thanks.

(PS... I just read your god post lmao.  Ok, it all makes sense to me now lol.  )

OH! GREAT ONE! who is only able to write in gutter level English and has the gall to insult the intellect of someone he knows nothing about will in future be ignored.

http://www.thenakedscientists.com/forum/index.php?topic=66954.0
Title: Re: If Energy is neither created nor used up, where did energy come from?
Post by: PmbPhy on 15/06/2016 07:06:55

Energy is not a physical thing but an attribute of mass. Mass itself is not straightforward. John uses the term mass without specifying its type. Is it rest mass, inertial mass, gravitational mass or relativistic mass? These distinctions are important and are the exact type of subtleties that John show by his own words not to understand. It is too easy to take on board misconceptions and to believe that they are accepted science. It is a minefield for the layman. If in doubt question what you read and ask for other opinions. The best answers will come from moderators.
Jeff is correct in what he said. When physicists use the term energy that is precisely what it means. Have you never read the Feynman Lectures on this topic? On the meaning and subject of energy Feynman writes
[quotes]
It is important to realize that in physics today, we have no knowledge of what energy is. We do not have a picture that energy comes in little blobs of a definite amount. It is not that way. However, there are formulas for calculating some numerical quantity, and we add it all together it gives “28” -  always the same number. It is an abstract thing in that it does not tell us the mechanism or the reasons for the various formulas.
The same sentiment is reflected in most good texts on physics such as those by French, Glashow, etc.
Title: Re: If Energy is neither created nor used up, where did energy come from?
Post by: Alan McDougall on 15/06/2016 08:00:12

Energy is not a physical thing but an attribute of mass. Mass itself is not straightforward. John uses the term mass without specifying its type. Is it rest mass, inertial mass, gravitational mass or relativistic mass? These distinctions are important and are the exact type of subtleties that John show by his own words not to understand. It is too easy to take on board misconceptions and to believe that they are accepted science. It is a minefield for the layman. If in doubt question what you read and ask for other opinions. The best answers will come from moderators.
Jeff is correct in what he said. When physicists use the term energy that is precisely what it means. Have you never read the Feynman Lectures on this topic? On the meaning and subject of energy Feynman writes
[quotes]
It is important to realize that in physics today, we have no knowledge of what energy is. We do not have a picture that energy comes in little blobs of a definite amount. It is not that way. However, there are formulas for calculating some numerical quantity, and we add it all together it gives “28” -  always the same number. It is an abstract thing in that it does not tell us the mechanism or the reasons for the various formulas.
The same sentiment is reflected in most good texts on physics such as those by French, Glashow, etc.

Feynman's the great physicists, comment that there are no little "Blobs" of energy nicely answers the question , because his little blob quote informs us that energy it is "Not"  A 'THING" that we find or will ever find in nature. Sadly to no avail because people, still seek it here, they seek it there, they seek the irritating illusive blob of non-existing  energy everywhere. Sadly for them, in vain, because the"blob of energy" does not exist as a real thing in nature, but a describable mathematical quality of matter and energy that is useful tool in the physics .
Title: Re: If Energy is neither created nor used up, where did energy come from?
Post by: IAMREALITY on 15/06/2016 15:07:44
OH! GREAT ONE! who is only able to write in gutter level English and has the gall to insult the intellect of someone he knows nothing about will in future be ignored.
Gutter level english lmao.  And awwww, how cute, you're gonna ignore me for merely coming at you with logic, could you be any more childish?  How bout actually responding to the points given to you instead of constantly replying as if you're the be all end all with your nose up in the air while obviously having no idea what you're talking about?  You are just too funny!  And nah, not insulting your intellect...  You've done more than a fine enough job of that on your own with your own words and sentiments son...

Oh, and ps... I will say I appreciate you calling me Great One though....  I don't mind sharing that title with Gretzky one bit!
Title: Re: If Energy is neither created nor used up, where did energy come from?
Post by: IAMREALITY on 15/06/2016 16:09:30

Jeff is correct in what he said. When physicists use the term energy that is precisely what it means. Have you never read the Feynman Lectures on this topic? On the meaning and subject of energy Feynman writes
It is important to realize that in physics today, we have no knowledge of what energy is. We do not have a picture that energy comes in little blobs of a definite amount. It is not that way. However, there are formulas for calculating some numerical quantity, and we add it all together it gives “28” -  always the same number. It is an abstract thing in that it does not tell us the mechanism or the reasons for the various formulas.
The same sentiment is reflected in most good texts on physics such as those by French, Glashow, etc.

Who is Feynman??


Just kidding hehehe.  I'm familiar with him of course and have watched some of his lectures, but not necessarily what you are referring to.  I did just read a little about it, however. 

I guess what throws me off is how based on what I know everything had come from a singularity of unimaginably hot, pure, higher state energy, and how mass and matter only came thereafter.  How can something only be an attribute of something else that does not yet exist?  I guess that's the part that throws me off a bit.

And was Jeff earlier inferring that it is only an attribute of mass or that right now that's merely the only way we have to describe it?
Title: Re: If Energy is neither created nor used up, where did energy come from?
Post by: JohnDuffield on 15/06/2016 17:15:50
You assume to know what Einstein meant. You think that your one opinion outweighs the multitude of professionals working directly with the particles whose energy to presume to know all about.
Einstein said what he said. "The mass of a body is a measure of its energy-content" (https://www.fourmilab.ch/etexts/einstein/E_mc2/www/). Don't blame me if some particle physicist tell you something that flatly contradicts Einstein.

The subtleties of science elude you John. You are like the proverbial bull shopping for china.
They don't and I'm not. There is no subtlety to E=mc². What Einstein said is there in black and white for all the world to see: "If a body gives off the energy L in the form of radiation, its mass diminishes by L/c²." If somebody tells you something different, don't just take it for granted, ask him why he's contradicting Einstein. And when he can't or won't explain, you'll know there's a problem, won't you?

Energy is not a physical thing but an attribute of mass.
No it isn't. Please don't peddle such twaddle.

Mass itself is not straightforward.
Mass is straightforward.

John uses the term mass without specifying its type.
When used without qualification, we mean rest mass.

Is it rest mass, inertial mass, gravitational mass or relativistic mass? These distinctions are important and are the exact type of subtleties that John show by his own words not to understand.
I understand them all. The latter three are nowadays considered to be measures of energy. A photon has a non-zero inertial mass, but it has a zero rest mass.

It is too easy to take on board misconceptions and to believe that they are accepted science.
That's what you've been doing. Energy is not an attribute of mass because the mass of a body is a measure of its energy-content.   

It is a minefield for the layman. If in doubt question what you read and ask for other opinions. The best answers will come from moderators.
The best answers come from people who explain things carefully and back up what they say with robust references to papers and evidence.
Title: Re: If Energy is neither created nor used up, where did energy come from?
Post by: Alan McDougall on 15/06/2016 17:28:54
You assume to know what Einstein meant. You think that your one opinion outweighs the multitude of professionals working directly with the particles whose energy to presume to know all about.
Einstein said what he said. "The mass of a body is a measure of its energy-content" (https://www.fourmilab.ch/etexts/einstein/E_mc2/www/). Don't blame me if some particle physicist tell you something that flatly contradicts Einstein.

The subtleties of science elude you John. You are like the proverbial bull shopping for china.
They don't and I'm not. There is no subtlety to E=mc². What Einstein said is there in black and white for all the world to see: "If a body gives off the energy L in the form of radiation, its mass diminishes by L/c²." If somebody tells you something different, don't just take it for granted, ask him why he's contradicting Einstein. And when he can't or won't explain, you'll know there's a problem, won't you?

Energy is not a physical thing but an attribute of mass.
No it isn't. Please don't peddle such twaddle.

Mass itself is not straightforward.
Mass is straightforward.

John uses the term mass without specifying its type.
When used without qualification, we mean rest mass.

Is it rest mass, inertial mass, gravitational mass or relativistic mass? These distinctions are important and are the exact type of subtleties that John show by his own words not to understand.
I understand them all. The latter three are nowadays considered to be measures of energy. A photon has a non-zero inertial mass, but it has a zero rest mass.

It is too easy to take on board misconceptions and to believe that they are accepted science.
That's what you've been doing. Energy is not an attribute of mass because the mass of a body is a measure of its energy-content.   

It is a minefield for the layman. If in doubt question what you read and ask for other opinions. The best answers will come from moderators.
The best answers come from people who explain things carefully and back up what they say with robust references to papers and evidence.

Richard Feynman the great physicists, comment that there are no little "Blobs" of energy, nicely answers the question , because his little blob quote informs us that energy it is "Not"  A 'THING" that we find or will ever find in nature.

Sadly to no avail because people, still seek it here, they seek it there, they seek the irritating illusive blob of non-existing energy everywhere. Sadly for them, in vain, because the"blob of energy" does not exist as a real thing in nature, but a describable mathematical quality of matter, that is useful tool in the physics and thermodynamics .

Title: Re: If Energy is neither created nor used up, where did energy come from?
Post by: IAMREALITY on 15/06/2016 17:46:04
Richard Feynman the great physicists, comment that there are no little "Blobs" of energy, nicely answers the question , because his little blob quote informs us that energy it is "Not"  A 'THING" that we find or will ever find in nature.

Sadly to no avail because people, still seek it here, they seek it there, they seek the irritating illusive blob of non-existing energy everywhere. Sadly for them, in vain, because the"blob of energy" does not exist as a real thing in nature, but a describable mathematical quality of matter, that is useful tool in the physics and thermodynamics .

De Ja Vu.  Not sure you're really saying much of anything at all though, or at least to the point that it needs to be repeated now for any further inquiry on the subject...  I can't help wondering if you're thinking it's more profound than it actually is?  But anyway, there may not be blobs of energy roaming about, but I'm not certain that makes it any less worthy of existence than fields, or waves, or anything else there aren't blobs of.  What I've gathered so far, is that energy is simply something we have not yet wrapped our heads around to the point of truly understanding, much like so many other aspects of physics, quantum physics, and everything else universal.  In the end it could have mind blowing explanations, and be far more 'real' than we could even begin to surmise right now.  The most important thing Feynman said was that we simply do not know what it is.  I would caution you'd be well served to not try to imply that you in fact do.
Title: Re: If Energy is neither created nor used up, where did energy come from?
Post by: jeffreyH on 16/06/2016 00:36:29
You assume to know what Einstein meant. You think that your one opinion outweighs the multitude of professionals working directly with the particles whose energy to presume to know all about.
Einstein said what he said. "The mass of a body is a measure of its energy-content" (https://www.fourmilab.ch/etexts/einstein/E_mc2/www/). Don't blame me if some particle physicist tell you something that flatly contradicts Einstein.

The subtleties of science elude you John. You are like the proverbial bull shopping for china.
They don't and I'm not. There is no subtlety to E=mc². What Einstein said is there in black and white for all the world to see: "If a body gives off the energy L in the form of radiation, its mass diminishes by L/c²." If somebody tells you something different, don't just take it for granted, ask him why he's contradicting Einstein. And when he can't or won't explain, you'll know there's a problem, won't you?

Energy is not a physical thing but an attribute of mass.
No it isn't. Please don't peddle such twaddle.

Mass itself is not straightforward.
Mass is straightforward.

John uses the term mass without specifying its type.
When used without qualification, we mean rest mass.

Is it rest mass, inertial mass, gravitational mass or relativistic mass? These distinctions are important and are the exact type of subtleties that John show by his own words not to understand.
I understand them all. The latter three are nowadays considered to be measures of energy. A photon has a non-zero inertial mass, but it has a zero rest mass.

It is too easy to take on board misconceptions and to believe that they are accepted science.
That's what you've been doing. Energy is not an attribute of mass because the mass of a body is a measure of its energy-content.   

It is a minefield for the layman. If in doubt question what you read and ask for other opinions. The best answers will come from moderators.
The best answers come from people who explain things carefully and back up what they say with robust references to papers and evidence.

Define what you think Einstein meant by radiation. You have your audience.
Title: Re: If Energy is neither created nor used up, where did energy come from?
Post by: jeffreyH on 16/06/2016 01:11:26
John, for your personal education and development. Read and absorb.  [;D]

https://en.m.wikipedia.org/wiki/Einstein_coefficients (https://en.m.wikipedia.org/wiki/Einstein_coefficients)
Title: Re: If Energy is neither created nor used up, where did energy come from?
Post by: Alan McDougall on 16/06/2016 02:10:52
John, for your personal education and development. Read and absorb.  [;D]

https://en.m.wikipedia.org/wiki/Einstein_coefficients (https://en.m.wikipedia.org/wiki/Einstein_coefficients)


I read the article. Thanks

I know you know what I am saying below? However, I would like to put it over in my own way!

Energy for instant is released from an atom when an electron changes is discrete position from either a higher or lower state from the electron cloud or "Orbit", ("which is not a correct description of the make up of an atom but useful analogy") with the resulting release of a photon of light "containing potential energy", as is in the case of atomic fusion in the sun, where hydrogen atoms in the core are progressively over vast periods of time fuse into heaver and heaver elements. This energy then gets released in turn into the universe increasing its state of chaos and entropy.

In the almost unimaginably distant future at say "the heat death of the universe", all the original energy released into the universe "would still be it is confines", but so dissipated and the entropic state increased to almost infinity, that no further activities could ever take place again.

The basic causes of release of energy is the fusion taking place due to the huge force of gravity and colossal temperature at the core of the sun Thus; in my opinion while gravity plays a huge part in the energy flow or entropy of the universe at large, "it not really the primordial source of energy".

Energy can only be described as "an equation of thermodynamics" and is not a real separate, tangible material thing that could, "hypothetically, be picked up and held in a persons hands"
Title: Re: If Energy is neither created nor used up, where did energy come from?
Post by: PmbPhy on 16/06/2016 08:06:47
Regarding energy and mass, I'd like to make a point that people might know subconsciously but not be aware of it consciously. Consider a  body whose relativistic mass m. Then The E in the expression E = mc2 is not the total energy of the body but what I like to refer to as the inertial energy which is the sum of the bodies rest energy and its kinetic energy. The E does not contain the energy of position, i.e., potential energy.

One of the excuses people use against relativistic mass is We don't need relativistic mass because it's the same thing as energy. This is wrong because energy means total energy which includes potential energy. However the E in  E = mc2 is not total energy because it doesn't contain potential energy.
Title: Re: If Energy is neither created nor used up, where did energy come from?
Post by: Alan McDougall on 16/06/2016 10:31:25
Regarding energy and mass, I'd like to make a point that people might know subconsciously but not be aware of it consciously. Consider a  body whose relativistic mass m. Then The E in the expression E = mc2 is not the total energy of the body but what I like to refer to as the inertial energy which is the sum of the bodies rest energy and its kinetic energy. The E does not contain the energy of position, i.e., potential energy.

One of the excuses people use against relativistic mass is We don't need relativistic mass because it's the same thing as energy. This is wrong because energy means total energy which includes potential energy. However the E in  E = mc2 is not total energy because it doesn't contain potential energy.

What then in the case of an matter - antimatter "Clash"? Does that release all the energy (Total)  "held"? in those two opposing forms of matter, converting them into gamma rays? However, in the event just described, "all of the energy"? has just been dissipated into the gamma ray cloud only changed in form.

We are lucky this did not happen in the early universe or we would not be debating this subject, especially in light of the fact that the universe would have then existed as a vast gamma-ray void.

There is no such thing as anti-energy, as some have speculated , energy is energy, the release and consequence usage thereof, can only be reflected as an equation of thermodynamics, in the macro world. At the quantum level it seems to be more mysterious and difficult to define?

There must be a loss somewhere, because it is impossible to convert 100% of energy from one form of it to another. where would that energy have gone?
Title: Re: If Energy is neither created nor used up, where did energy come from?
Post by: PmbPhy on 16/06/2016 11:15:21
Quote from: Alan McDougall
What then in the case of an matter - antimatter "Clash"? Does that release all the energy (Total)  "held"? in those two opposing forms of matter, converting them into gamma rays? However, in the event just described, "all of the energy"? has just been dissipated into the gamma ray cloud only changed in form.
The phrase release the energy is a confusing one since it doesn't have a exact meaning. Typically when people use this phrase they have in mind that photons are energy that's been released or something similar. However that's thinking of photons as "being" energy rather than "having" energy, which is the correct viewpoint. I recommend avoiding that kind of phrasing. It can lead to the wrong idea. For example when a nuke goes off, a lot of the damage that the radiation (in the forum or photons, alpha rays, beta rays, neutrons etc) that is released can do is in the form of particles with very high kinetic energy. So perhaps that's what you may have meant when you say released, i.e. particles with kinetic energy are released?

What is not well known is that when a particle annihilates its antiparticle the result is not always photons. Sometimes its other particles.

Quote from: Alan McDougall
There must be a loss somewhere, because it is impossible to convert 100% of energy from one form of it to another. where would that energy have gone?
It's possible to convert potential energy completely into kinetic energy. It's also possible to convert rest energy completely into radiant energy (in the form of photons).
Title: Re: If Energy is neither created nor used up, where did energy come from?
Post by: jeffreyH on 16/06/2016 12:59:49
Einstein said what he said. "The mass of a body is a measure of its energy-content" (https://www.fourmilab.ch/etexts/einstein/E_mc2/www/). Don't blame me if some particle physicist tell you something that flatly contradicts Einstein.

So you are effectively ignoring the gamma factor in there which shows that it is relativistic mass that Einstein is talking about. If you knew your physics you wouldn't make such silly mistakes.

Title: Re: If Energy is neither created nor used up, where did energy come from?
Post by: timey on 16/06/2016 13:25:17
Regarding energy and mass, I'd like to make a point that people might know subconsciously but not be aware of it consciously. Consider a  body whose relativistic mass m. Then The E in the expression E = mc2 is not the total energy of the body but what I like to refer to as the inertial energy which is the sum of the bodies rest energy and its kinetic energy. The E does not contain the energy of position, i.e., potential energy.

One of the excuses people use against relativistic mass is We don't need relativistic mass because it's the same thing as energy. This is wrong because energy means total energy which includes potential energy. However the E in  E = mc2 is not total energy because it doesn't contain potential energy.

Ah - interesting!

Ok - so let me get with this one a bit better.

You say E=mc2 is inclusive of what you call 'inertial energy' and kinetic energy.

KE is calculated as 0.5mv2=KE

So when I see the formula E=mc2, can I assume that the calculation for KE has already been completed?

And that the m in the equation is already complete with the relativistic mass added via the additional KE energy?

And... is this what distinguishes the terms of E=mc2 and e=mc2?
Title: Re: If Energy is neither created nor used up, where did energy come from?
Post by: JohnDuffield on 16/06/2016 13:32:06
Richard Feynman the great physicists, comment that there are no little "Blobs" of energy, nicely answers the question , because his little blob quote informs us that energy it is "Not"  A 'THING" that we find or will ever find in nature.
I'm sorry Alan, but Feynman was at odds with Einstein there. I'm something of a "Feynman fan", but on this, I'm with Einstein.   

Sadly to no avail because people, still seek it here, they seek it there, they seek the irritating illusive blob of non-existing energy everywhere. Sadly for them, in vain, because the"blob of energy" does not exist as a real thing in nature, but a describable mathematical quality of matter, that is useful tool in the physics and thermodynamics.
Energy exists. It is not merely some mathematical quality of matter.


What then in the case of an matter - antimatter "Clash"? Does that release all the energy (Total)  "held"? in those two opposing forms of matter, converting them into gamma rays?
Yes.

We are lucky this did not happen in the early universe or we would not be debating this subject, especially in light of the fact that the universe would have then existed as a vast gamma-ray void.
Gamma-gamma pair production is the reverse of annihilation.

There is no such thing as anti-energy, as some have speculated , energy is energy, the release and consequence usage thereof, can only be reflected as an equation of thermodynamics, in the macro world. At the quantum level it seems to be more mysterious and difficult to define?
It isn't mysterious at all.

There must be a loss somewhere, because it is impossible to convert 100% of energy from one form of it to another. where would that energy have gone?
There is no loss. Energy is the one thing you can neither create nor destroy. 

Energy can only be described as "an equation of thermodynamics" and is not a real separate, tangible material thing that could, "hypothetically, be picked up and held in a persons hands".
Energy is real. It isn't tangible like a suitcase atom bomb is tangible. You can't hold energy in your hands like you can hold a  suitcase atom bomb in your hands. But when the latter detonates and energy is released, you will realise that energy is very real indeed. For about a nanosecond.

The bottom line is this: matter is made of it. You are made of it. 
Title: Re: If Energy is neither created nor used up, where did energy come from?
Post by: JohnDuffield on 16/06/2016 13:54:40
...However the E in  E = mc2 is not total energy because it doesn't contain potential energy.
I'm afraid it does. Consider a brick in front of you on a shelf. E=mc² applies. Now you push the brick off the shelf, whereupon some of its potential energy is converted into kinetic energy as it falls. The brick hits the floor, whereupon the kinetic energy is dissipated. Conservation of energy applies, along with the mass deficit (https://en.wikipedia.org/wiki/Binding_energy#Mass-energy_relation). The brick is again at rest, and E=mc² still applies. But now the rest-mass-energy of the brick is less than what it was. When you throw that brick back up onto the shelf you do work on it. You add energy to it. You increase its mass. Note that the Earth is involved in this process, but not much. Momentum is shared equally, but energy is not. Google on collision bullet block (https://www.google.co.uk/?gfe_rd=cr&ei=EUM_VY2-Juiq8wfv_oCgDA&gws_rd=ssl#q=collision+bullet+block).

...that is released can do is in the form of particles with very high kinetic energy
This is a popscience particle-physics view that is at odds with the E=hc/λ wave nature of photons, and at odds with what Einstein said. Radiation does not consist of billiard-ball particles. It consists of photons, and a photon is a wave. When you remove the kinetic energy from the wave, the wave no longer exists. The same is not true for a billiard ball. This is why Einstein said radiation is a form of energy.

Title: Re: If Energy is neither created nor used up, where did energy come from?
Post by: PmbPhy on 16/06/2016 14:16:30
Quote from: timey
You say E=mc2 is inclusive of what you call 'inertial energy' and kinetic energy.
No. What I said is that Inertial Energy = Rest Energy + Kinetic energy

Quote from: timey
KE is calculated as 0.5mv2=KE
That's the low speed approximation to the kinetic energy, K. Let E0 = m0c2 = the rest energy of the particle where m0 is the proper mass, aka rest mass, of the particle. Let gamma = 1/sqrt{1 - v2/c2}. The expression for the relativistic kinetic energy is then given by K = (gamma - 1)m0c2 = (gamma - 1)E0. If you have a strong math background and would like to follow the derivation for this expression then you can find it on my website at: http://www.newenglandphysics.org/physics_world/sr/work_energy.htm

When following the derivation it's helpful to keep in mind that the kinetic energy is defined by the requirement that the change in kinetic energy equal the work done by the external force acting on the particle. This is called the Work-Energy Theorem.

With all of this we can find the expression for the inertial energy E.

E = K + E0 = (gamma - 1)E0 + E0 = gamma*E0

or

E = gamma*m0c2 = mc2

where m = gamma*m0 = the relativistic mass of the particle.

Quote from: timey
So when I see the formula E=mc2, can I assume that the calculation for KE has already been completed?
Yes, in the sense that I just demonstrated.

Quote from: timey
And that the m in the equation is already complete with the relativistic mass added via the additional KE energy?
The m in E = mc2 is the relativistic mass. I don't know what you mean by "added via the additional KE energy."
Title: Re: If Energy is neither created nor used up, where did energy come from?
Post by: Alan McDougall on 16/06/2016 14:18:03
Richard Feynman the great physicists, comment that there are no little "Blobs" of energy, nicely answers the question , because his little blob quote informs us that energy it is "Not"  A 'THING" that we find or will ever find in nature.
I'm sorry Alan, but Feynman was at odds with Einstein there. I'm something of a "Feynman fan", but on this, I'm with Einstein.   

Sadly to no avail because people, still seek it here, they seek it there, they seek the irritating illusive blob of non-existing energy everywhere. Sadly for them, in vain, because the"blob of energy" does not exist as a real thing in nature, but a describable mathematical quality of matter, that is useful tool in the physics and thermodynamics.
Energy exists. It is not merely some mathematical quality of matter.


What then in the case of an matter - antimatter "Clash"? Does that release all the energy (Total)  "held"? in those two opposing forms of matter, converting them into gamma rays?
Yes.

We are lucky this did not happen in the early universe or we would not be debating this subject, especially in light of the fact that the universe would have then existed as a vast gamma-ray void.
Gamma-gamma pair production is the reverse of annihilation.

There is no such thing as anti-energy, as some have speculated , energy is energy, the release and consequence usage thereof, can only be reflected as an equation of thermodynamics, in the macro world. At the quantum level it seems to be more mysterious and difficult to define?
It isn't mysterious at all.

There must be a loss somewhere, because it is impossible to convert 100% of energy from one form of it to another. where would that energy have gone?
There is no loss. Energy is the one thing you can neither create nor destroy. 

Energy can only be described as "an equation of thermodynamics" and is not a real separate, tangible material thing that could, "hypothetically, be picked up and held in a persons hands".
Energy is real. It isn't tangible like a suitcase atom bomb is tangible. You can't hold energy in your hands like you can hold a  suitcase atom bomb in your hands. But when the latter detonates and energy is released, you will realise that energy is very real indeed. For about a nanosecond.

The bottom line is this: matter is made of it. You are made of it. 

I meant "where has the energy gone" because there is never a 100% conversion of energy from its one form into another it was wrong of me to say "lost" However, you are wrong matter is not a block of energy it is something that contains potential energy as you, yourself pointed out.

Energy is just a mathematical expression of how work is transferred from one body to another and is not a physical material thing at all period!

I wonder if the Higgs Boson  could fit into this debate?

Alan
Title: Re: If Energy is neither created nor used up, where did energy come from?
Post by: PmbPhy on 16/06/2016 14:33:20
Quote from: Alan McDougall
I meant "where has the energy gone" because there is never a 100% conversion of energy from its one form into another...
On what basis did you arrive at that conclusion? Consider an harmonic oscillator.  The energy of it is given by E = K + V. As the system, e.g. a pendulum, oscillates from one its top position to its bottom position and back starting the cycle over again, the energy shuffles back and forth from potential energy to kinetic energy with a complete conversion of one to the other. I gave another example using pair annihilation of an electron and a positron. Assuming that the particles start from rest and are in contact they will annihilate producing two photons thus converting rest energy into the radiant energy of the two photons. The energy of a photon is 100% kinetic energy so the conversion is between rest energy and kinetic energy. If the particles start off moving towards each other then the system will start off with some kinetic energy. The result will be the same with all of the rest energy being changed to kinetic energy which is part of the final kinetic energy of the two photons. If you place a vane into a bucket of water and let the vane have an initial kinetic energy then the system will eventually settle down with all of the kinetic energy being changed to thermal energy.
Title: Re: If Energy is neither created nor used up, where did energy come from?
Post by: timey on 16/06/2016 15:15:01
Quote from: timey
You say E=mc2 is inclusive of what you call 'inertial energy' and kinetic energy.
No. What I said is that Inertial Energy = Rest Energy + Kinetic energy

Quote from: timey
KE is calculated as 0.5mv2=KE
That's the low speed approximation to the kinetic energy, K. Let E0 = m0c2 = the rest energy of the particle where m0 is the proper mass, aka rest mass, of the particle. Let gamma = 1/sqrt{1 - v2/c2}. The expression for the relativistic kinetic energy is then given by K = (gamma - 1)m0c2 = (gamma - 1)E0. If you have a strong math background and would like to follow the derivation for this expression then you can find it on my website at: http://www.newenglandphysics.org/physics_world/sr/work_energy.htm

When following the derivation it's helpful to keep in mind that the kinetic energy is defined by the requirement that the change in kinetic energy equal the work done by the external force acting on the particle. This is called the Work-Energy Theorem.

With all of this we can find the expression for the inertial energy E.

E = K + E0 = (gamma - 1)E0 + E0 = gamma*E0

or

E = gamma*m0c2 = mc2

where m = gamma*m0 = the relativistic mass of the particle.

Quote from: timey
So when I see the formula E=mc2, can I assume that the calculation for KE has already been completed?
Yes, in the sense that I just demonstrated.

Quote from: timey
And that the m in the equation is already complete with the relativistic mass added via the additional KE energy?
The m in E = mc2 is the relativistic mass. I don't know what you mean by "added via the additional KE energy."

Pete - thanks, although I'm struggling to follow, I am following and would ask that you explain gamma a little.

I left school age 11 and am self taught in everything.  I don't have a formal basis in maths beyond long division but have been studying and catch on real quick.  I understand the principle of maths in relation to formula and geometry, (albeit that this understanding is of a visualist-ic nature), but am having trouble with relativistic mass, and the accumulative tendencies of relativistic mass and gravity.

So if e=mc2 relates to inertial energy and rest mass.  Then I assume that KE is then added as a calculation of 0.5 of that rest mass times velocity squared. (which is what I meant by adding KE to the equation).  For e=mc2 to be inclusive of KE, the result of 0.5mv2 is converted into mass and added to the rest mass to become the 'new' m for the E=mc2 equation...(this is what I suspect your maths are telling me, am I close?)

Then...when potential energy is considered, does this change relativistic mass again?
Title: Re: If Energy is neither created nor used up, where did energy come from?
Post by: Alan McDougall on 16/06/2016 16:07:33
Quote from: Alan McDougall
I meant "where has the energy gone" because there is never a 100% conversion of energy from its one form into another...
On what basis did you arrive at that conclusion? Consider an harmonic oscillator.  The energy of it is given by E = K + V. As the system, e.g. a pendulum, oscillates from one its top position to its bottom position and back starting the cycle over again, the energy shuffles back and forth from potential energy to kinetic energy with a complete conversion of one to the other. I gave another example using pair annihilation of an electron and a positron. Assuming that the particles start from rest and are in contact they will annihilate producing two photons thus converting rest energy into the radiant energy of the two photons. The energy of a photon is 100% kinetic energy so the conversion is between rest energy and kinetic energy. If the particles start off moving towards each other then the system will start off with some kinetic energy. The result will be the same with all of the rest energy being changed to kinetic energy which is part of the final kinetic energy of the two photons. If you place a vane into a bucket of water and let the vane have an initial kinetic energy then the system will eventually settle down with all of the kinetic energy being changed to thermal energy.

My wordings always seem to lack clarity this is what I meant below.

In the practical application of energy conversion at the electricity generating company where I worked for most of my life, there was a constant need for scheduled maintenance of the huge boiler and generators at the Power Station to keep them at maximum efficiency, because wear and tear over time resulted in poorer and poorer energy conversion of this open system and a drop in the efficiency of the machines.

At the most ideal situation the best efficiency of such as a system was in the region of 38%. The rest of the energy being dissipate or wasted out of the cooling towers, condensers and smoke stacks etc, etc.

"Thus the energy had gone somewhere" and this is the core of what I meant when I stated that energy conversion is never 100%. If this were possible we could create a perpetual motion machine .

Through, the conversion of thermal energy into other forms,we reduce the temperature of the system, increasing its entropy, keeping its efficiency at much less than 100%" (even when energy is not allowed to escape from the system).

This is because thermal energy has already been partly spread out among many available states of a collection of microscopic and macro, particles constituting the system.

In such circumstances, a measure of entropy, or dissipation, dictates that future states of an isolated system, must be of at least equal evenness in energy distribution.

In other words, there is no way to concentrate energy without spreading out energy somewhere else, and that is that I what I really meant in my badly worded response?

Regards

Alan.
Title: Re: If Energy is neither created nor used up, where did energy come from?
Post by: JohnDuffield on 16/06/2016 18:18:14
I meant "where has the energy gone" because there is never a 100% conversion of energy from its one form into another.
There always is. Energy is wasted in an engine or machine, but none of it ever actually disappears. Energy conservation always applies.

However, you are wrong matter is not a block of energy it is something that contains potential energy as you, yourself pointed out.
That potential energy is actually "hidden" kinetic energy, internal to the body. Gravity converts some of this internal kinetic energy into external kinetic energy. When this external kinetic energy is dissipated, you're left with a mass deficit.   

Energy is just a mathematical expression of how work is transferred from one body to another and is not a physical material thing at all period!
Energy isn't just some mathematical expression. Physical material things are made of it. That's the message from Einstein and E=mc² (https://www.fourmilab.ch/etexts/einstein/E_mc2/www/), and unless somebody has a good explanation as to why Einstein is wrong, I'm sticking with Einstein. I recommend that you do the same.

I wonder if the Higgs Boson  could fit into this debate?
Yes of course. The Higgs boson is quite literally made from the kinetic energy given to the protons in the LHC. It's not totally unlike creating an electron and a positron out of photon kinetic energy in pair production. Note this on Matt Strassler's blog (https://profmattstrassler.com/2012/10/23/does-the-higgs-field-give-the-higgs-particle-its-mass-or-not/): "The Higgs field, though it provides the mass for all other known particles with masses, does not provide the Higgs particle with its mass".
Title: Re: If Energy is neither created nor used up, where did energy come from?
Post by: PmbPhy on 16/06/2016 18:40:51
Quote from: timey
Pete - thanks, although I'm struggling to follow, I am following and would ask that you explain gamma a little.
gamma is a Greek letter which is set equal to 1/sqrt{1 - v^2/c^2}. It's just a letter which is used to make the formulas look simpler.

Quote from: timey
Then I assume that KE is then added as a calculation of 0.5 of that rest mass times velocity squared.
As I explained in reply #82 that's the non-relativistic expression for kinetic energy. It's only valid for velocities which are much smaller than the speed of light.

Quote from: timey
(which is what I meant by adding KE to the equation).  For e=mc2 to be inclusive of KE, the result of 0.5mv2 is converted into mass and added to the rest mass to become the 'new' m for the E=mc2 equation...(this is what I suspect your maths are telling me, am I close?)
No.

Quote from: timey
Then...when potential energy is considered, does this change relativistic mass again?
As I explained above potential energy, i.e. the energy associated with the position of a particle in a field, is not part of the energy in E = mc^2. However, when a particle is in such a field it is subject to a force. That force changes the speed of the particle which in turn changes the relativistic mass of the particle.
Title: Re: If Energy is neither created nor used up, where did energy come from?
Post by: jeffreyH on 16/06/2016 18:54:21
Timey think of gamma in a similar way to a percentage. Although it isn't it may be useful to think of it in those terms. If we take a stationary object to have 100% mass then if we cause it to move and increase its speed we have to add some percentage to the mass proportional to the increase in speed. The faster the speed the larger the 'percentage' increase. The nearer to light speed the speed of our object is the nearer to an infinite amount of mass it has. Near to infinity however is misleading in this case since defining how near to infinity anything is is not possible. All that can be said is that the greater the increase in the percentage of mass the harder it is to move the object even faster. I have avoided time dilation on purpose here as it wouldn't help. That is a simplified interpretation which hopefully will clarify things for you.
Title: Re: If Energy is neither created nor used up, where did energy come from?
Post by: agyejy on 16/06/2016 19:07:21
As I explained above potential energy, i.e. the energy associated with the position of a particle in a field, is not part of the energy in E = mc^2.

That is not strictly true. The clearest example is the fact that a collection of unbound neutrons and protons has more mass than the same collection bound into a nucleus (assuming the binding energy of the nucleus is negative). Calculating that difference in mass requires that you take into account any electromagnetic forces as well as the attraction of the strong force (and probably the weak force if you want a lot of precision). Going down another layer the mass of the quarks in the protons and neutrons are a very small fraction of the mass of the protons and neutrons. The rest of the mass comes primarily from the fields of the strong force modified again by electromagnetic interactions between the quarks (and probably the weak force to some extent otherwise beta decay is hard to explain). Most of the invariant mass of composite particles like protons and neutrons usually comes from the force fields holding them together (i.e. potential energy). Even a hydrogen atom is slightly (very very slightly) less massive than the combined unbound masses of a proton and an electron.  Even for a single free electron there is still a mass renormalization due to the interaction of the electron with its own field. This is often described as the electron being surrounded by a collection of virtual particles that increase the observed mass.

Title: Re: If Energy is neither created nor used up, where did energy come from?
Post by: timey on 16/06/2016 19:42:28
Timey think of gamma in a similar way to a percentage. Although it isn't it may be useful to think of it in those terms. If we take a stationary object to have 100% mass then if we cause it to move and increase its speed we have to add some percentage to the mass proportional to the increase in speed. The faster the speed the larger the 'percentage' increase. The nearer to light speed the speed of our object is the nearer to an infinite amount of mass it has. Near to infinity however is misleading in this case since defining how near to infinity anything is is not possible. All that can be said is that the greater the increase in the percentage of mass the harder it is to move the object even faster. I have avoided time dilation on purpose here as it wouldn't help. That is a simplified interpretation which hopefully will clarify things for you.

Thanks Jeff - it's s good analogy, one that I've heard before and have grasped the understanding of.

But... to understand 'exactly' how these masses and energies add up mathematically within relativity is my goal.

I am just a tad unclear as to what part of the physical process 'gamma' actual 'is', hence my request.

And my interest, for clarity, lies in the notion that a rock lying on the ground has a rest mass and inertial energy.  It is made up of atoms that have rest mass and inertial energy.  Presumably these atoms have a corresponding De Broglie matter wave.  Pick the rock up and we must add potential energy.  Throw the rock and we must add kinetic energy...

Does the frequency of the atoms that are the matter of the rock increase, and the matter wave decrease with the additional potential energy?

Does the frequency of the atoms that are the matter of the rock increase, and the matter wave decrease with the addition of kinetic energy?
Title: Re: If Energy is neither created nor used up, where did energy come from?
Post by: timey on 16/06/2016 19:55:52
gamma is a Greek letter which is set equal to 1/sqrt{1 - v^2/c^2}. It's just a letter which is used to make the formulas look simpler.

Pete - I do not understand the physicality of 1/square root 1 etc, and what this term relates to within the physical process.  m is mass, v is velocity, e is energy of various types, squaring is a mathematical process, where does the 1 originate from?  What is 1?

As I explained in reply #82 that's the non-relativistic expression for kinetic energy. It's only valid for velocities which are much smaller than the speed of light.

Are you saying that the mathematical process of calculating KE changes above a certain velocity?  If so, what physical process causes the mathematical proportionality of relativistic and non-relativistic velocities to be un-reconcilable within the same mathematical process?
Title: Re: If Energy is neither created nor used up, where did energy come from?
Post by: jeffreyH on 16/06/2016 20:11:36
Firstly let's look at v and c that form the fraction v/c that is used in gamma. The value of v can never equal c but must be less than c at all times. We can look at this as v being a percentage of c. In this way we can multiply v by a fraction to represent this. When v = 1/2*c it is half  the speed of light and when v = 99/100*c it is 99% the speed of light etc. So that if v = 1/2*c this is like saying v/c = 1/2.

Since the fraction used in gamma is v^2/c^2 then for the value of 1/2 this becomes 1^2/2^2 which gives 1/4. This is not the end of it though because gamma has the square root of 1 - v^2/c^2 as the denominator. In this case we need to find the square root of 1-1/4. So then we are looking at the square root of 3/4 which approximates to 0.866. The final step is 1/0.866 which translates to a value of 1.1547 approx. So our mass is increased in this case by 115.5% approx at half light speed. If anybody sees an error in my working please point it out.

This is the mathematical description. The physical causes are an entirely different matter. Find that and you will be famous.
Title: Re: If Energy is neither created nor used up, where did energy come from?
Post by: Alan McDougall on 16/06/2016 20:32:48
Firstly let's look at v and c that form the fraction v/c that is used in gamma. The value of v can never equal c but must be less than c at all times. We can look at this as v being a percentage of c. In this way we can multiply v by a fraction to represent this. When v = 1/2*c it is half  the speed of light and when v = 99/100*c it is 99% the speed of light etc. So that if v = 1/2*c this is like saying v/c = 1/2.

Since the fraction used in gamma is v^2/c^2 then for the value of 1/2 this becomes 1^2/2^2 which gives 1/4. This is not the end of it though because gamma has the square root of 1 - v^2/c^2 as the denominator. In this case we need to find the square root of 1-1/4. So then we are looking at the square root of 3/4 which approximates to 0.866. The final step is 1/0.866 which translates to a value of 1.1547 approx. So our mass is increased in this case by 115.5% approx at half light speed. If anybody sees an error in my working please point it out.

This is the mathematical description. The physical causes are an entirely different matter. Find that and you will be famous.

All that is informative but what does it have to do with the question of the thread "If Energy is not created or destroyed where does it come from"?
Title: Re: If Energy is neither created nor used up, where did energy come from?
Post by: jeffreyH on 16/06/2016 22:30:11
The increases in relativistic mass/energy via the gamma function indicate that some fundamental interaction is at work at the quantum level. Due to these effects being tied very closely to those of gravitation then finding the cause will lead you towards an answer to your question.That is if it can be answered at all.
Title: Re: If Energy is neither created nor used up, where did energy come from?
Post by: Alan McDougall on 16/06/2016 22:40:30
The increases in relativistic mass/energy via the gamma function indicate that some fundamental interaction is at work at the quantum level. Due to these effects being tied very closely to those of gravitation then finding the cause will lead you towards an answer to your question.That is if it can be answered at all.

I agree!; if the answer is ever found it would come from the quantum level, with gravitation playing some vital role in the process
Title: Re: If Energy is neither created nor used up, where did energy come from?
Post by: Alan McDougall on 16/06/2016 23:22:32
This might be of interest?

http://energy.nobelprize.org/presentations/linde.pdf
Title: Re: If Energy is neither created nor used up, where did energy come from?
Post by: PmbPhy on 17/06/2016 02:42:30
Quote from: agyejy
That is not strictly true. The clearest example is the fact that a collection of unbound neutrons and protons has more mass than the same collection bound into a nucleus (assuming the binding energy of the nucleus is negative).
Hi agyejy. You're incorrect. What I said is precisely true. Please reread it carefully. This time please keep in mind the difference between the potential energy of position of a particle in a field and the internal potential energy of a body due to the interactions between the constituents of the body, i.e. internal potential energy. Wolfgang Rindler explains this in his text Relativity; Special, General and Cosmological - 2nd Ed., (2006). I did you a favor and extracted section 6.3 which is entitled The Equivalence of Mass and Energy. You can download it from my website at: http://www.newenglandphysics.org/other/Rindler_on_potential_energy_and_mass.pdf

On page 113 Rindler writes
Quote
One kind of energy that does not contribute to mass is potential energy of position.
Perhaps if you read it in the context of the entire section it will make more sense to you. Rindler is much better at explaining these things than I am.  [:)]

And, yes. I'm well-aware of the fact that the internal potential energy of a closed system contributes to the mass of the system. I took that into account when I described nuclear fission on my website. See:
http://www.newenglandphysics.org/physics_world/sr/nuclear_fission.htm

The problem here is that we have different notions of potential energy in mind. In the post above where I explained this I said that it was the potential energy of position that doesn't contribute to mass energy. You appear to have confused that with the internal potential energies between the particles that make up that body.

To be precise, if there is a charged particle, such as an electron, moving in a static electric field then the total energy of the electron is the sum of three forms of energy: kinetic energy, rest mass energy and potential energy. An electron doesn't have internal potential energy . The total energy, which I'll label W, is the sum of those three energies, i.e. W = K + E0+ U where U is the potential energy associated with position.

My intention in raising this point is because I believed that some people might not know this fact.
Title: Re: If Energy is neither created nor used up, where did energy come from?
Post by: PmbPhy on 17/06/2016 04:21:55
Quote from: Alan McDougall
All that is informative but what does it have to do with the question of the thread "If Energy is not created or destroyed where does it come from"?
As I attempted to explain in my first post in this thread, the energy needn't come from anywhere. We can think of it as always having had he value of zero. We can do this because the absolute value of the total energy in the universe doesn't have a physical meaning. What does have a physical meaning is changes in each form of energy. As I'm sure you know the absolute value of the potential energy of a particle, or system of particles, is not physically meaningful. Only gradients in the potential energy function is physically meaningful. Recall that the potential energy of a particle, V, in a field of force F is defined as F = -8319f01cae0e5752ba2e719544659959.gifV. So you can add any constant to V without changing the force on the particle. So you can always set the value of the total energy to zero.

Alan Guth explains something very similar to all of this in his book The Inflationary Universe in Chapter 17 which is entitled Universe Ex Nihilo which starts on page 271. The idea was first speculated by Edward Tryon in a paper which appeared in Nature in 1973. Tyron explained it in terms of the fact that the gravitational energy is negative while mass-energy is positive. Tyron learned from the well-known general relativist Peter Bergmann that in any closed universe the negative gravitational energy cancels the energy of matter exactly. The total energy, or as Guth writes "or equivalently the total mass" is precisely equal to zero.

I believe that answers your question. I'd be more than happy to place chapter 17 of that book into a PDF file and upload it onto my website if anybody would like to read the entire chapter. If so then please let me know.
Title: Re: If Energy is neither created nor used up, where did energy come from?
Post by: agyejy on 17/06/2016 07:15:33
Hi agyejy. You're correct, but then again so am I.

Unfortunately that isn't really true.

Quote
Yes. I'm well-aware of the fact that the internal potential energy of a closed system contributes to the mass of the system. I took that into account when I described nuclear fission on my website. See:
http://www.newenglandphysics.org/physics_world/sr/nuclear_fission.htm

The reason that we're both correct is because we're each correctly talking about two different things. I'm talking about the external  potential energy of a body while you're talking about the internal potential energies between the particles that make up that body.

I direct your attention to the couple of sentences of my previous post:

Quote from: Me
Even for a single free electron there is still a mass renormalization due to the interaction of the electron with its own field. This is often described as the electron being surrounded by a collection of virtual particles that increase the observed mass.

There is no experimental evidence for an internal structure to electrons and physicists have been looking for awhile. However, you cannot correctly calculate the mass of a totally free completely isolated electron without taking into account interactions between the electron and its own fields. This is the so called self-energy of the electron and the processes of correcting for it is known as mass renormalization.

Beyond that the concepts of internal and external are completely arbitrary and assigned by humans. Speaking in terms of physics and calculations you should get the same result regardless of if you pick a single electron as your frame or reference or the entire atom. Which frame results in easier mathematics is largely dependent on the properties you wish to calculate. For example, for calculating most of the properties of an atom using the center of mass frame (which really just amounts to keeping the nucleus stationary due to how much heavier it is) results in much simpler math. That doesn't mean you can't do the same calculations in an arbitrary frame it just wouldn't be as convenient. The spin-orbit interaction is a case where it is often easier to consider the frame of the electron and treat the nucleus as a source of an external magnetic field that interacts with the intrinsic magnetic moment of the electron.

You are correct in making the distinction that the mass of a system is only well defined for a closed system but there is no real point in making a distinction between internal and external. We only see changes in mass due to changes in potential energy when that change in energy is carried away (i.e. the system is not closed). The simplest example of this would be the emission of a photon either from an electron moving to a state of lower potential energy or similarly a proton in the nucleus moving to a state of lower potential energy (this usually gives us a photon in the gamma ray energy range). In fact the very reason we don't see a change in mass when potential energy is converted into kinetic energy is precisely because both forms of energy impact the measurement of mass in identical ways.

Quote
Here's what I'm talking about: If there is a charged particle, such as an electron, which is moving in a static electric field then the electron will have three types of energy: Kinetic energy, rest mass energy and potential energy. An electron does not have any internal structure so there will not be any internal potential energy associated with interacting parts. The total energy, which I'll label W, is the sum of those three energies, i.e. W = K + E0+ U where U is the potential energy associated with the position of the electron in the electric field. The potential energy, U, of the electron in the electric field is U = eV where e is the charge of the electron and V is the electric potential.

http://arxiv.org/pdf/0709.4427.pdf <- Dynamical electron mass in a strong magnetic field

The jist of it is that the mass of an electron depends on the strength of the magnetic field at its current position.

Quote
There is an important difference here to keep in mind and that's the difference between the potential energy of a particle due to its position in a field and the internal potential energy between particles within a body. I was speaking of the former which you appear to have confused it with the later. My intention in raising this point was because I believed that some people might have these two things confused.

A distinction between what is and isn't within a body is completely arbitrary and has no impact on physical observables. You can freely change your definition of a system from encompassing the entire Earth to a single atom on the Earth any it will have no impact on the measured properties of that atom. Another way of putting it is that non-zero fields (i.e. fields that create differences in potential energy) always have a source. Given a non-zero field one can always find a way to extend the definition of the current system to include the source of the non-zero field thereby making it internal. Thus it makes absolutely no sense to distinguish the action of internal fields from external fields because all fields capable of doing anything (i.e. non-zero fields) have a source that can be made internal upon changing the arbitrary definition of the boundaries of the system under consideration.
Title: Re: If Energy is neither created nor used up, where did energy come from?
Post by: Alan McDougall on 17/06/2016 07:47:02
Considering the amount of energy packed in the nucleus of a single uranium atom, or the energy that has been continuously radiating from the sun for billions of years, or the fact that there are 10^80 particles in the observable universe, it seems that the total energy in the universe must be an inconceivably vast quantity. But it's not; it's probably zero.

Light, matter and antimatter are what physicists call "positive energy." And yes, there's a lot of it (though no one is sure quite how much). Most physicists think, however, that there is an equal amount of "negative energy" stored in the gravitational attraction that exists between all the positive-energy particles. The positive exactly balances the negative, so, ultimately, there is no energy in the universe at all.
Title: Re: If Energy is neither created nor used up, where did energy come from?
Post by: IAMREALITY on 17/06/2016 17:38:29
so, ultimately, there is no energy in the universe at all.

That's an extremely flawed conclusion.  There's plenty of it.  If you had 10 particles of matter and 10 particles of anti-matter, and they were separate from each other, then even though if they came in contact they'd obliterate each other, would it be accurate to then state, whilst they are still separate and minding their business, that there are actually no particles at all?
Title: Re: If Energy is neither created nor used up, where did energy come from?
Post by: Alan McDougall on 17/06/2016 20:04:18
so, ultimately, there is no energy in the universe at all.

That's an extremely flawed conclusion.  There's plenty of it.  If you had 10 particles of matter and 10 particles of anti-matter, and they were separate from each other, then even though if they came in contact they'd obliterate each other, would it be accurate to then state, whilst they are still separate and minding their business, that there are actually no particles at all?
[/s]

To the entity that responded to my posts you are not worthy of my attention or the attention of any other logical thinking person and I ignore all your rubbish!
Title: Re: If Energy is neither created nor used up, where did energy come from?
Post by: IAMREALITY on 17/06/2016 21:23:24
so, ultimately, there is no energy in the universe at all.

That's an extremely flawed conclusion.  There's plenty of it.  If you had 10 particles of matter and 10 particles of anti-matter, and they were separate from each other, then even though if they came in contact they'd obliterate each other, would it be accurate to then state, whilst they are still separate and minding their business, that there are actually no particles at all?
[/s]

To the entity that responded to my posts you are not worthy of my attention or the attention of any other logical thinking person and I ignore all your rubbish!

How bout you  address the logic and content of the post itself? Or do you simply not have it in you? Fact is, what I posted was perfectly clear and a very straightforward analogy refuting your point. Do you have an aversion to straightforward logic or something?
Title: Re: If Energy is neither created nor used up, where did energy come from?
Post by: PmbPhy on 18/06/2016 06:53:50
Quote from: agyejy
Unfortunately that isn't really true.
Oh, great! Another newbie who thinks he knows everything but can't fathom the simple statement from Rindler's SR text, i.e.
Quote from: Wolfgang Rindler
One kind of energy that does not  contribute to mass is potential energy of position.
As we physicists say It's intuitively obvious even to the most casual observer!  [;D]  Anybody who thinks about this carefully will understand this fact. For instance; if there is an electron in a uniform static electric field then the potential energy of the electron will depend on where the particle is in the field. However the force on the electron is independent of where the electron is so there is no physical difference between the electron when its at positions of differing potentials. Therefore there can be no difference in the mass. Got it???

Of course you're welcome to write an article and attempt to publish it in the American Journal of Physics and attempt to argue that you're right and experiments are wrong. By the way. You're abusing the meaning of mass renormalization. From what you wrote its clear that you have a poor understanding of it. Every single physicist worth his salt and every single relativist that's existed since SR was created knows this very trivial fact of SR. 

But in one sense you're right. We're not both right. I'm right and you're wrong and that's according to observations made in the lab during experiments with particle accelerators. As I said, the potential energy due to the position of a particle in an external field does not effect the mass of the particle. That's a fact that's demonstrated every single time that a particle accelerator is in operation. If you don't understand that then you don't understand mass-energy equivalence. Your comment on mass renormalization is completely irrelevant to this point. You really need to better. That's why I suggested that you read the section from Rindler that I gave you. But if you can't even understand that then I don't see the point of me trying to help you understand this anymore than I already have.
Title: Re: If Energy is neither created nor used up, where did energy come from?
Post by: Alan McDougall on 18/06/2016 08:35:03
so, ultimately, there is no energy in the universe at all.

That's an extremely flawed conclusion.  There's plenty of it.  If you had 10 particles of matter and 10 particles of anti-matter, and they were separate from each other, then even though if they came in contact they'd obliterate each other, would it be accurate to then state, whilst they are still separate and minding their business, that there are actually no particles at all?
[/s]

To the entity that responded to my posts you are not worthy of my attention or the attention of any other logical thinking person and I ignore all your rubbish!

How bout you  address the logic and content of the post itself? Or do you simply not have it in you? Fact is, what I posted was perfectly clear and a very straightforward analogy refuting your point. Do you have an aversion to straightforward logic or something?

I love straightforward logic but and do not have an aversion to it , however, I have an aversion to you as a person, "which is my right" and all your "terminological inexacititudes"
Title: Re: If Energy is neither created nor used up, where did energy come from?
Post by: agyejy on 18/06/2016 09:29:55
Oh, great! Another newbie who thinks he knows everything but can't fathom the simple statement from Rindler's SR text, i.e.
Quote from: Wolfgang Rindler
One kind of energy that does not  contribute to mass is potential energy of position.

I would ask that you don't make assumptions about what I know and my level of education. I happen to think that is somewhat less than polite. I managed to avoid impugning your education and ability to reason and I would appreciate the same consideration.

Also I think it is important look at the entire quote:

Quote from: Wolfgang Rindler
One kind of energy that does not contribute to mass is potential energy of position. In classical mechanics, a particle moving in an electromagnetic (or gravitational)
field is often said to possess potential energy, so that the sum of its kinetic and
potential energies remains constant. This is a useful ‘book-keeping’ device, but energy
conservation can also be satisfied by debiting the field with an energy loss equal to the
kinetic energy gained by the particle. In relativity there are good reasons for adopting
the second alternative,
though the first can be used as an occasional shortcut: the ‘real’
location of any part of the energy is no longer a mere convention, since energy (as
mass) gravitates; that is, it contributes measurably (in principle) to the curvature of
spacetime at its location.

Basically Rindler is of the opinion that potential energy is simply a "'book-keeping' device" and feels that it is better to attribute all potential energy to the field. However, the changes in the external field are intrinsically linked to the particle and its position in the field. So regardless of where you claim the energy is stored you still have to know where the particle is to know how the energy density changes and the place where the energy density changes the most from the presence of the particle has to be the position of the particle. You might ascribe the potential energy to the field but if you go out a measure a particle in the field the measurement you get is always going to include both the particle and the field together.

Further this is the opinion of someone who is an authority in relativity not in quantum mechanics. Relativity has nothing at all to say about the origin of mass and on its own does a very poor job of describing the quantum realm. If you want to discuss the origin of mass and how mass is impacted by the presence of fields you need to be using a relativistic quantum field theory at the bare minimum. Relativity alone is either unable to answer the question or is just plain wrong.

Quote
As we physicists say It's intuitively obvious even to the most casual observer!  [;D]  Anybody who thinks about this carefully will understand this fact. For instance; if there is an electron in a uniform static electric field then the potential energy of the electron will depend on where the particle is in the field. However the force on the electron is independent of where the electron is so there is no physical difference between the electron when its at positions of differing potentials. Therefore there can be no difference in the mass. Got it???

Force is not mass. Force is related to mass by the proportionality constant we call acceleration or more specifically by the time derivative of momentum. A physicist should be well aware that acceleration is not a fixed quantity and that a constant force on a changing mass results in a changing acceleration. Of course that point is made entirely moot by the point Rindler himself raised. Namely that the sum of potential energy and kinetic energy in a conservative force field (for anyone else that might not know off the top of their head a uniform static electric field is a conservative field) doesn't change. Therefore if you accept that the potential energy and the kinetic energy both contribute to the mass the only conclusion is that in a conservative force field mass does not change at least as long as we're sticking to only relativity.

Quote
Of course you're welcome to write an article and attempt to publish it in the American Journal of Physics and attempt to argue that you're right and experiments are wrong. By the way. You're abusing the meaning of mass renormalization. From what you wrote its clear that you have a poor understanding of it. Every single physicist worth his salt and every single relativist that's existed since SR was created knows this very trivial fact of SR.

Again I point out that we're talking about a region of physics were relativity on its own is known to be wrong. I would hope any relativist would be able to recognize that fact. As far as papers about the effect of fields on particle masses (not written by me but still a paper):

http://arxiv.org/pdf/0709.4427.pdf <- Dynamical electron mass in a strong magnetic field

And if you don't like that one you can look through any of the references which say very similar things. These should be a fairly good start:

Quote
[10] V. P. Gusynin, V. A. Miransky, and I. A. Shovkovy, Phys.
Rev. D 52, 4747 (1995); Nucl. Phys. B462, 249 (1996);
D. K. Hong, Y. Kim, and S. J. Sin, Phys. Rev. D 54,
7879 (1996).
[11] D.-S. Lee, C. N. Leung, and Y. J. Ng, Phys. Rev. D 55,
6504 (1997); E. J. Ferrer and V. de la Incera, ibid. 58,
065008 (1998).
[12] V. P. Gusynin, V. A. Miransky, and I. A. Shovkovy,
Phys. Rev. Lett. 83, 1291 (1999); Nucl. Phys. B563, 361
(1999); Phys. Rev. D 67, 107703 (2003); A. V. Kuznetsov
and N. V. Mikheev, Phys. Rev. Lett. 89, 011601 (2002).
[13] V. P. Gusynin, V. A. Miransky, and I. A. Shovkovy,
Phys. Rev. Lett. 90, 089101 (2003); A. V. Kuznetsov
and N. V. Mikheev, ibid., 089102 (2003).
[14] C. N. Leung and S.-Y. Wang, Nucl. Phys. B747, 266
(2006); Ann. Phys. (N.Y.) 322, 701 (2007).

As for your opinion of my understanding of mass renormalization you'll understand if I don't trust your judgement given that you didn't immediately realize that there was no way the electric field of a free photon should have more energy than the magnetic field and the fact that you seem to have at best very sparse knowledge of relativistic quantum field theories especially in regards to what they say about the origins of mass.

Quote
But in one sense you're right. We're not both right. I'm right and you're wrong and that's according to observations made in the lab during experiments with particle accelerators. As I said, the potential energy due to the position of a particle in an external field does not effect the mass of the particle. That's a fact that's demonstrated every single time that a particle accelerator is in operation. If you don't understand that then you don't understand mass-energy equivalence.

Nothing about the operation of a particle accelerator does any such thing. Simply put the fields used in a particle accelerator aren't remotely strong enough to have a noticeable effect above all the other sources of beam instabilities and sources for mass changes in the beams (particles are picked up by and lost from the acceleration bunches all the time). The reasons the fields aren't strong enough are actually pretty simple. The first is that at least for the magnets we don't have electromagnets capable of sustained fields much higher than what is used and the second is what is known as rf breakdown which can set in at electric field gradients of less than 100 MV/m. Though currently prototype plasma wakefield generators are doing much better by well using plasmas and also lasers.

Quote
Your comment on mass renormalization is completely irrelevant to this point. You really need to better.

I simply cannot fathom why anyone would think that a discussion of how we calculate the masses of elementary particles and how those calculations are changed in the presence of strong magnetic fields is unrelated to a discussion about the masses of things.

Quote
That's why I suggested that you read the section from Rindler that I gave you. But if you can't even understand that then I don't see the point of me trying to help you understand this anymore than I already have.

As I pointed out above Rindler himself admits that as far as classical physics is concerned neither of the viewpoints he discusses are correct. Furthermore, he didn't say that it was wrong to think in terms of potential energy. What he said was that he feels that when working specifically with relativity and only relativity there are reasons to use his other viewpoint. Clearly there is no reason to believe Rindler's views
should hold for quantum mechanics or relativistic quantum field theories. Rindler definitely has no real expertise in terms of discussing the origins of mass because his area of expertise is in a theory in which the existence of mass is axiomatic. Certainly he could be more of an expert than me but he probably isn't more of an expert than people that publish about relativistic quantum field theories or those that do work on the standard model.

Rindler's quote was nothing more than him pushing his opinion about how people should think about potential energy when working with pure relativity. It should never be mistaken for a universal fact or even a fact in terms of relativity. The viewpoint expressed by Rindler is simply his choice of pedagogy and he is entitled to that choice. I doubt he would support it as unassailable scientific fact.
Title: Re: If Energy is neither created nor used up, where did energy come from?
Post by: Alan McDougall on 18/06/2016 09:51:07
It still is apparent that some of us still seek Richards Feynman blobs of energy with others insisting that energy is just a mathematical equation, used in physics and thermodynamics, as a means of explaining the conversion of work (Energy) from one form into the other.?

Or

Is there some "Primordial Source" of from which all energy emerged, out from, something like an infinitely tight spring that was sprung at the big bang or earlier.

Or

Do we exist as part of an infinite multiverse of which our universe is just one example of, and if so we are left with the problem of "Infinite Regression"

Alan
Title: Re: If Energy is neither created nor used up, where did energy come from?
Post by: PmbPhy on 18/06/2016 10:05:28
Quote from: agyejy
I would ask that you don't make assumptions about what I know and my level of education.
I made no assumption about your background in math and physics. I was speaking about your response which is what I like to call word salad which means that it's all words with no direct bearing on the case at hand. I.e. mass renormalization has absolutely nothing to do with the fact that potential energy of position does not contribute to the mass of an object. And your comments on virtual particles is also irrelevant. First of all that has to do with quantum mechanics and this is all about classical physics. The virtual particles that you're talking about only pertain to the screening effect of charges. Mass renormalization only has to do with the relationship between bare mass and rest mass.

I clearly said Oh, great! Another newbie who thinks he knows everything but can't fathom the simple statement from Rindler's SR text, ... and that's quite true.  I don't care if you have a college degree in both math and physics. It's your reasoning that's all wrong here. It's as if you're taking words that you found in textbooks which refer to mass and attempting to fit them into what you (erroneously) believe to be the case. It's your lack of your ability to understand and follow arguments, by Rindler and myself, which clearly shows that you think that you know different but in reality you're clearly wrong. I explained why you were wrong by giving you an illustrative example and you totally ignored it which shows that you can't correctly argue your case and don't understand the reasons why you're wrong, i.e. you still don't get it. At this point its very clear to me that you just don't have enough of an open mind to understand this or you can't admit you're wrong so you won't consider the arguments which prove it. However I do care what others here think so for their sake I'll explain it again in more detail, for their sake.

An electron does not have a rest mass that can be changed, neither by a change in position, a change in kinetic energy or its position in an electric field, i.e. it's invariant - period. In the case of kinetic energy the relativistic mass depends on kinetic energy. However it doesn't depend on the potential energy of its position. This is a very simple fact. This doesn't mean that the electrons potential energy can't change. It means that when it does change both the relativistic mass and rest mass remain the same. Consider again the following  example: let there be a Cartesian coordinate system in the inertial frame of reference S. Also in S let there be a uniform electric field pointing in the x-direction. The choice of zero potential is arbitrary so there's no physical way to distinguish when the potential energy of the particle is zero, where its positive or where it's negative. Also, all physical spatial locations in S are physically identical. It's quite literally impossible to determine where you are in the field since its spatially uniform. However when the position of the electron is changed from z = 0 to z = h the potential energy undergoes a change in potential energy by an amount 967878d1da852d4b07a961e3168b0fff.gifU = e967878d1da852d4b07a961e3168b0fff.gifV where V is the electric potential. Recall that changes in potential energy are physically meaningful. However there is no physical difference in the electron between the first and second case. Therefore there can be no difference in the rest mass of the electron. Especially since it's impossible to change the rest mass of the electron.

Your misunderstanding of what Rindler wrote in his text cannot change the fact that - One kind of energy that does not  contribute to mass is potential energy of position.   What part of that you're claiming is wrong is something you've never stated, for what reason nobody can say. But it's very suspicious that you didn't.

Re - Rindler is of the opinion that potential energy is simply a "'book-keeping' device"

So what? Any good physicist knows that to be true. In fact that's what I explained in my website at:
http://www.newenglandphysics.org/physics_world/cm/what_is_energy.htm
Quote
How seriously must we take the physical existence of this energy? No more and no less than any other bookkeeping practices. The physical quantities are those such as the EM field.
Here's a challenge that I'll wager that you'll never accept for fear of being wrong and having to admit that you made a mistake. That seems to be the case here. E-mail Rindler. Please do the forum a favor. Instead of passing on your misconception please send him an e-mail explaining to him why his statement One kind of energy that does not  contribute to mass is potential energy of position. is wrong. Tell him that Peter Brown was trying to explain to you why this is a perfectly correct statement and that your interpretation, with all your erroneous comments about mass renormalization is wrong. If you'd like I'd be happy to explain what mass renormalization is exactly and why you used it incorrectly, if you ask politely.  [:)]

However if you don't do this then I'll assume that you have no intention of getting straight to the truth and will place you in my ignore list. I only say this so you won't waste time asking me questions and then making invalid assumptions about why I don't respond.
Title: Re: If Energy is neither created nor used up, where did energy come from?
Post by: garth john greiner on 18/06/2016 10:43:07
The only response I can give to this question is that energy is built in to the chemistry of the universe.
I feel that energy is generated by the behavior and interactions of the diverse chemicals that exist in the universe,
Ie: all the different particles that exist have a job to do and by doing so interact with each other and thus generating energy.
The force of the big bang ignited this process.
Title: Re: If Energy is neither created nor used up, where did energy come from?
Post by: garth john greiner on 18/06/2016 10:49:46
It makes you wonder what sort of energy production was present pre big bang to initiate that level of force but that's the golden question we would all like to know !!! Lol
Title: Re: If Energy is neither created nor used up, where did energy come from?
Post by: JohnDuffield on 18/06/2016 16:42:25
...Anybody who thinks about this carefully will understand this fact. For instance; if there is an electron in a uniform static electric field then the potential energy of the electron will depend on where the particle is in the field. However the force on the electron is independent of where the electron is so there is no physical difference between the electron when its at positions of differing potentials. Therefore there can be no difference in the mass.
All:beware, this is wrong I'm afraid. Check out the mass deficit (https://en.wikipedia.org/wiki/Binding_energy#Mass-energy_relation). When you pull an electron away from a proton, you do work on it. You add energy to it. You increase its mass. The mass of the hydrogen atom is less than the mass of the free proton plus the mass of the free electron.   

Title: Re: If Energy is neither created nor used up, where did energy come from?
Post by: jeffreyH on 18/06/2016 17:25:31
...Anybody who thinks about this carefully will understand this fact. For instance; if there is an electron in a uniform static electric field then the potential energy of the electron will depend on where the particle is in the field. However the force on the electron is independent of where the electron is so there is no physical difference between the electron when its at positions of differing potentials. Therefore there can be no difference in the mass.
All:beware, this is wrong I'm afraid. Check out the mass deficit (https://en.wikipedia.org/wiki/Binding_energy#Mass-energy_relation). When you pull an electron away from a proton, you do work on it. You add energy to it. You increase its mass. The mass of the hydrogen atom is less than the mass of the free proton plus the mass of the free electron.

Hold your horses John Boy there's too many chickens in the coop. The loss in energy is due to the negative binding energy. Note I didn't say mass.
Title: Re: If Energy is neither created nor used up, where did energy come from?
Post by: agyejy on 18/06/2016 17:56:27
I made no assumption about your background in math and physics. I was speaking about your response which is what I like to call word salad which means that it's all words with no direct bearing on the case at hand. I.e. mass renormalization has absolutely nothing to do with the fact that potential energy of position does not contribute to the mass of an object. And your comments on virtual particles is also irrelevant. First of all that has to do with quantum mechanics and this is all about classical physics. The virtual particles that you're talking about only pertain to the screening effect of charges. Mass renormalization only has to do with the relationship between bare mass and rest mass.

I simply cannot fathom why anyone would think that a discussion of how we calculate the masses of elementary particles and how those calculations are changed in the presence of strong magnetic fields is unrelated to a discussion about the masses of things.

http://arxiv.org/pdf/0709.4427.pdf <- Dynamical electron mass in a strong magnetic field (i.e. the rest mass is changed due to the effect of the field on the screening)

Quote
[10] V. P. Gusynin, V. A. Miransky, and I. A. Shovkovy, Phys.
Rev. D 52, 4747 (1995); Nucl. Phys. B462, 249 (1996);
D. K. Hong, Y. Kim, and S. J. Sin, Phys. Rev. D 54,
7879 (1996).
[11] D.-S. Lee, C. N. Leung, and Y. J. Ng, Phys. Rev. D 55,
6504 (1997); E. J. Ferrer and V. de la Incera, ibid. 58,
065008 (1998).
[12] V. P. Gusynin, V. A. Miransky, and I. A. Shovkovy,
Phys. Rev. Lett. 83, 1291 (1999); Nucl. Phys. B563, 361
(1999); Phys. Rev. D 67, 107703 (2003); A. V. Kuznetsov
and N. V. Mikheev, Phys. Rev. Lett. 89, 011601 (2002).
[13] V. P. Gusynin, V. A. Miransky, and I. A. Shovkovy,
Phys. Rev. Lett. 90, 089101 (2003); A. V. Kuznetsov
and N. V. Mikheev, ibid., 089102 (2003).
[14] C. N. Leung and S.-Y. Wang, Nucl. Phys. B747, 266
(2006); Ann. Phys. (N.Y.) 322, 701 (2007).

Quote
I clearly said Oh, great! Another newbie who thinks he knows everything but can't fathom the simple statement from Rindler's SR text, ... and that's quite true.  I don't care if you have a college degree in both math and physics. It's your reasoning that's all wrong here. It's as if you're taking words that you found in textbooks which refer to mass and attempting to fit them into what you (erroneously) believe to be the case. It's your lack of your ability to understand and follow arguments, by Rindler and myself, which clearly shows that you think that you know different but in reality you're clearly wrong. I explained why you were wrong by giving you an illustrative example and you totally ignored it which shows that you can't correctly argue your case and don't understand the reasons why you're wrong, i.e. you still don't get it. At this point its very clear to me that you just don't have enough of an open mind to understand this or you can't admit you're wrong so you won't consider the arguments which prove it. However I do care what others here think so for their sake I'll explain it again in more detail, for their sake.

Well that's nothing but an outright lie. Since when is an entire paragraph:

Quote from: My response
Force is not mass. Force is related to mass by the proportionality constant we call acceleration or more specifically by the time derivative of momentum. A physicist should be well aware that acceleration is not a fixed quantity and that a constant force on a changing mass results in a changing acceleration. Of course that point is made entirely moot by the point Rindler himself raised. Namely that the sum of potential energy and kinetic energy in a conservative force field (for anyone else that might not know off the top of their head a uniform static electric field is a conservative field) doesn't change. Therefore if you accept that the potential energy and the kinetic energy both contribute to the mass the only conclusion is that in a conservative force field mass does not change at least as long as we're sticking to only relativity.

ignoring something?

Quote
An electron does not have a rest mass that can be changed, neither by a change in position, a change in kinetic energy or its position in an electric field, i.e. it's invariant - period.

Again:

 http://arxiv.org/pdf/0709.4427.pdf <- Dynamical electron mass in a strong magnetic field (i.e. the rest mass is changed due to the effect of the field on the screening)

Quote
[10] V. P. Gusynin, V. A. Miransky, and I. A. Shovkovy, Phys.
Rev. D 52, 4747 (1995); Nucl. Phys. B462, 249 (1996);
D. K. Hong, Y. Kim, and S. J. Sin, Phys. Rev. D 54,
7879 (1996).
[11] D.-S. Lee, C. N. Leung, and Y. J. Ng, Phys. Rev. D 55,
6504 (1997); E. J. Ferrer and V. de la Incera, ibid. 58,
065008 (1998).
[12] V. P. Gusynin, V. A. Miransky, and I. A. Shovkovy,
Phys. Rev. Lett. 83, 1291 (1999); Nucl. Phys. B563, 361
(1999); Phys. Rev. D 67, 107703 (2003); A. V. Kuznetsov
and N. V. Mikheev, Phys. Rev. Lett. 89, 011601 (2002).
[13] V. P. Gusynin, V. A. Miransky, and I. A. Shovkovy,
Phys. Rev. Lett. 90, 089101 (2003); A. V. Kuznetsov
and N. V. Mikheev, ibid., 089102 (2003).
[14] C. N. Leung and S.-Y. Wang, Nucl. Phys. B747, 266
(2006); Ann. Phys. (N.Y.) 322, 701 (2007).

Do you know something the entire field of scientists working on relativistic field theories don't know?

Quote
In the case of kinetic energy the relativistic mass depends on kinetic energy. However it doesn't depend on the potential energy of its position. This is a very simple fact. This doesn't mean that the electrons potential energy can't change. It means that when it does change both the relativistic mass and rest mass remain the same. Consider again the following  example: let there be a Cartesian coordinate system in the inertial frame of reference S. Also in S let there be a uniform electric field pointing in the x-direction. The choice of zero potential is arbitrary so there's no physical way to distinguish when the potential energy of the particle is zero, where its positive or where it's negative. Also, all physical spatial locations in S are physically identical. It's quite literally impossible to determine where you are in the field since its spatially uniform. However when the position of the electron is changed from z = 0 to z = h the potential energy undergoes a change in potential energy by an amount 967878d1da852d4b07a961e3168b0fff.gifU = e967878d1da852d4b07a961e3168b0fff.gifV where V is the electric potential. Recall that changes in potential energy are physically meaningful. However there is no physical difference in the electron between the first and second case. Therefore there can be no difference in the rest mass of the electron. Especially since it's impossible to change the rest mass of the electron.

Again a uniform electric field is a conservative field so the sum of kinetic energy and potential energy must remain unchanged. Therefore either the kinetic energy must have changed by an amount exactly equal to the change in potential energy or some energy was added/removed from the system. In the case of the former since totally energy didn't change then clearly the relativistic mass didn't change. In the latter case some energy was clearly added/removed that had nothing to do with the original field. If you imagine electrons moving around without thinking about how the movement is accomplished you are going to come up with wrong answers. Of course this has very little to do with how the fields affect the mass renormalization of an electron by changing its self-energy and therefore its renormalized mass.

Quote
Your misunderstanding of what Rindler wrote in his text cannot change the fact that - One kind of energy that does not  contribute to mass is potential energy of position.   What part of that you're claiming is wrong is something you've never stated, for what reason nobody can say. But it's very suspicious that you didn't.

Again since when are paragraphs:

Quote from: My response
Basically Rindler is of the opinion that potential energy is simply a "'book-keeping' device" and feels that it is better to attribute all potential energy to the field. However, the changes in the external field are intrinsically linked to the particle and its position in the field. So regardless of where you claim the energy is stored you still have to know where the particle is to know how the energy density changes and the place where the energy density changes the most from the presence of the particle has to be the position of the particle. You might ascribe the potential energy to the field but if you go out a measure a particle in the field the measurement you get is always going to include both the particle and the field together.

Further this is the opinion of someone who is an authority in relativity not in quantum mechanics. Relativity has nothing at all to say about the origin of mass and on its own does a very poor job of describing the quantum realm. If you want to discuss the origin of mass and how mass is impacted by the presence of fields you need to be using a relativistic quantum field theory at the bare minimum. Relativity alone is either unable to answer the question or is just plain wrong.

Quote from: Also my response
As I pointed out above Rindler himself admits that as far as classical physics is concerned neither of the viewpoints he discusses are correct. Furthermore, he didn't say that it was wrong to think in terms of potential energy. What he said was that he feels that when working specifically with relativity and only relativity there are reasons to use his other viewpoint. Clearly there is no reason to believe Rindler's views
should hold for quantum mechanics or relativistic quantum field theories. Rindler definitely has no real expertise in terms of discussing the origins of mass because his area of expertise is in a theory in which the existence of mass is axiomatic. Certainly he could be more of an expert than me but he probably isn't more of an expert than people that publish about relativistic quantum field theories or those that do work on the standard model.

Rindler's quote was nothing more than him pushing his opinion about how people should think about potential energy when working with pure relativity. It should never be mistaken for a universal fact or even a fact in terms of relativity. The viewpoint expressed by Rindler is simply his choice of pedagogy and he is entitled to that choice. I doubt he would support it as unassailable scientific fact.

not stating something?

Quote
Re - Rindler is of the opinion that potential energy is simply a "'book-keeping' device"

So what? Any good physicist knows that to be true. In fact that's what I explained in my website at:
http://www.newenglandphysics.org/physics_world/cm/what_is_energy.htm
Quote
How seriously must we take the physical existence of this energy? No more and no less than any other bookkeeping practices. The physical quantities are those such as the EM field.

The entire statement was:

Quote
Basically Rindler is of the opinion that potential energy is simply a "'book-keeping' device" and feels that it is better to attribute all potential energy to the field.

Which was less clearly worded than maybe it could have been. I was not really saying anything against the notion of energy as a book-keeping device. I was simply summarizing Rindler's statements in order to point out that no matter where you say the energy is (in the particle or the field) it doesn't really change your observations just how you calculate what you should observe.

Quote
Here's a challenge that I'll wager that you'll never accept for fear of being wrong and having to admit that you made a mistake. That seems to be the case here. E-mail Rindler. Please do the forum a favor. Instead of passing on your misconception please send him an e-mail explaining to him why his statement One kind of energy that does not  contribute to mass is potential energy of position. is wrong. Tell him that Peter Brown was trying to explain to you why this is a perfectly correct statement and that your interpretation, with all your erroneous comments about mass renormalization is wrong. If you'd like I'd be happy to explain what mass renormalization is exactly and why you used it incorrectly, if you ask politely.  [:)]

I'm not doing this for several reasons. One is that I'm sure Rindler is far too busy to really care about this. Simply put it is a waste of his time and energy. I'm not going to bother a 92 year old man over this and even suggesting it is absurd. Furthermore, as I have already pointed out Rindler's area of expertise does not lie in relativistic quantum field theories and so he cannot be considered a valid expert on how mass renormalization (and therefore observed mass) changes in the presence of strong fields. However, it is instructive to look at how Matt Strassler thinks about energy and mass:

https://profmattstrassler.com/articles-and-posts/particle-physics-basics/mass-energy-matter-etc/the-energy-that-holds-things-together/

Quote
The ripples are quanta, or particles; they each have mass and motion energy, both of which are positive.

The green field’s disturbance has some energy too; it’s also positive, though often quite small compared to the energy of the particles in a case like this. That’s often called field energy.

But there is additional energy in the relationship between the various fields; where the blue and green fields are both large, there is energy, and where the green and orange fields are both large, there is also energy. And here’s the strange part. If you compare Figures 1 and 2, both of them have energy in the region where the blue and green fields are large. But the presence of the ripple in the orange field in the vicinity alters the green field, and therefore changes the energy in the region where the blue field’s ripple is sitting, as indicated in Figure 3.

Depending upon the details of how the orange and green fields interact with each other, and how the blue and green fields interact with each other, the change in the energy may be either positive or negative. This change is what I’m going to call interaction energy.

...

But now I want to bring you back to something I said in my mass and energy article, one of Einstein’s key insights that he obtained from working out the consequences of his equations. If you have a system of objects, the mass of the system is not the sum of the masses of the objects that it contains. It is not even proportional to the sum of the energies of the particles that it contains. It is the total energy of the system divided by c2, as viewed by an observer who is stationary relative to the system.  (For an observer relative to whom the system is moving, the system will have additional motion-energy, which does not contribute to the system’s mass.)  And that total energy involves

the mass energies of the particles (ripples in the fields), plus
the motion-energies of the particles, plus
other sources of field-energy from non-ripple disturbances, plus
the interaction energies among the fields.
What do we learn from the fact that the energy required to break apart hydrogen is 14 electron volts? Well, once you’ve broken the hydrogen atom apart you’re basically left with a proton and an electron that are far apart and not moving much. At that point, the energy of the system is

the mass energies of the particles  = electron mass-energy + proton mass-energy = 510, 999 electron-volts + 938,272,013 electron-volts
the motion-energies of the particles = 0
other sources of field-energy from non-ripple disturbances = 0
the interaction energies among the fields = 0
Meanwhile, we know that before we broke it up, the system of a hydrogen atom had energy that was 14 electron volts less than this.

Now the mass-energy of an electron is always 510, 999 electron-volts and the mass-energy of a proton is always 938,272,013 electron-volts, no matter what they are doing, so the mass-energy contribution to the total energy is the same for hydrogen as it is for a widely separated electron and proton.  What must be the case is that

the motion-energies of the particles inside hydrogen
PLUS other sources of field-energy from non-ripple disturbances (really really small here)
PLUS the interaction energies among the fields
MUST EQUAL the binding energy of -14 electron volts.
In fact, if you do the calculation, the way the numbers work out is (approximately)

the motion-energies of the particles = +14 electron volts
other sources of field-energy from non-ripple disturbances = really really small
the interaction energies among the fields = -28 electron volts.
and the sum of these things is -14 electron volts.

It’s not an accident that the interaction energy is -2 times the motion energy; roughly, that comes from having a 1/r2 force law for electrical forces. Experts: it follows from the virial theorem.

So today's particle physicists clearly have a different way of thinking about potential energy (called interaction energy) than Rindler does. I'm more inclined to trust them on this matter based on their expertise.

Quote
However if you don't do this then I'll assume that you have no intention of getting straight to the truth and will place you in my ignore list. I only say this so you won't waste time asking me questions and then making invalid assumptions about why I don't respond.

This is utterly unreasonable. There are many reasons I might not want to bug a 92 year old man about this. Some of them are given above. I've presented many different sources to back up my claims and demonstrated that the one source (aside from yourself) that you've used can't really be considered as an expert in what is currently under discussion. You've flat out ignored my sources up to this point going so far as saying I never made arguments which I very clearly made. From where I am sitting the person showing an unwillingness to get to the truth is you.
Title: Re: If Energy is neither created nor used up, where did energy come from?
Post by: PmbPhy on 18/06/2016 18:14:57
Quote from: agyejy
I simply cannot fathom why anyone would think that a discussion of how we calculate the masses of elementary particles and how those calculations are changed in the presence of strong magnetic fields is unrelated to a discussion about the masses of things.
That's clear proof that not only have you been unable to understand the subject that I've been telling you but that you're unwilling to learn it. Nobody has been talking about elementary particles except you. The fact is that it's totally unrelated to the subject at hand, i.e. that you cannot change the proper mass of a particle by potential energy of position. Once again you've demonstrated that you're unable to show that the example I gave is wrong and it's for that reason you keep ignoring it.

Quote from: agyejy
http://arxiv.org/pdf/0709.4427.pdf <- Dynamical electron mass in a strong magnetic field (i.e. the rest mass is changed due to the effect of the field on the screening)
This shows that you appear to be searching for articles which appear to be related to the subject when in fact they're unrelated. If an electron is placed into a magnetic the potential energy is zero and for that reason the proper mass cannot change for that reason.

As I said, you're shown that you're refusing to take the challenge of contacting Rindler to straighten out all of your misconceptions. I have no time for ignorant people such as yourself. That's why you're going into my ignore list. It's now clear to me and probably others, why you're hiding your formal background in math and physics. You don't have any. Instead you pick through papers which appear to support your bad ideas and use them as if they proved your point. They don't.

Again folks - Ignore this person. He's clueless.
Title: Re: If Energy is neither created nor used up, where did energy come from?
Post by: agyejy on 18/06/2016 19:13:40
That's clear proof that not only have you been unable to understand the subject that I've been telling you but that you're unwilling to learn it. Nobody has been talking about elementary particles except you. The fact is that it's totally unrelated to the subject at hand, i.e. that you cannot change the proper mass of a particle by potential energy of position. Once again you've demonstrated that you're unable to show that the example I gave is wrong and it's for that reason you keep ignoring it.

Let's take second to examine the logic of that last sentence. If i have demonstrated that I'm unable to show that your example is wrong than I must have attempted to show that it is wrong and failed (and I didn't fail). If I made an attempt to show that your example is wrong then clearly I read it and responded to it. If I read it and responded to it then how could that ever be considered ignoring by any reasonable definition of the word ignore?


Quote
This shows that you appear to be searching for articles which appear to be related to the subject when in fact they're unrelated. If an electron is placed into a magnetic the potential energy is zero and for that reason the proper mass cannot change for that reason.

As far as we have observed there are no spinless charged particles so all charged particles have a magnetic moment and therefore potential energy in a magnetic field. Additionally, any moving charged particle consitutes a current that has an associated magnetic field which means it also has a potential energy when subjected to an external magnetic field.

Quote
As I said, you're shown that you're refusing to take the challenge of contacting Rindler to straighten out all of your misconceptions.

This ignores all of the very good reasons I gave for not doing that. Not the least of which is that Rindler isn't even a proper expert for this discussion. I even gave an alternative more appropriate expert.

Quote
I have no time for ignorant people such as yourself.

This is just a straight insult.

Quote
That's why you're going into my ignore list.

I guess all that talk about getting to the truth was meaningless than. I've made solid points backed with varied references. You've got one reference that I've shown isn't even really a good one for this. I've also never once insulted you.

Quote
It's now clear to me and probably others, why you're hiding your formal background in math and physics. You don't have any.

Again an insult. Also, ignores other perfectly valid reasons like I don't want crazy people emailing me with crazy questions or messing with my life.

Quote
Instead you pick through papers which appear to support your bad ideas and use them as if they proved your point. They don't.

Saying it is so does not make it so. You have not even remotely supported your accusation and now you claim that you aren't even going to try. Ignore me if you want but anybody that has been reading this knows I had the stronger argument.

Quote
Again folks - Ignore this person. He's clueless.

And another insult to close things out. Honestly if anyone should be ignored for here it is you for your propensity to insult people for no other reason than that they disagree with you.
Title: Re: If Energy is neither created nor used up, where did energy come from?
Post by: agyejy on 18/06/2016 20:56:29
I feel I should take a momentum to clarify my position because I am actually arguing two different points and I feel that I'm doing a terrible job at keeping them separate. The two points are:

1) A quantity someone might call rest mass can be altered in the presence of very strong fields by impacting the virtual particle cloud that surrounds real particles. If the vacuum state around a particle changes the mass it picks up (or loses) due to interactions with the vacuum will change as well and there might not be an obvious reason. This would be the "other sources of field-energy from non-ripple disturbances" that Matt Strassler mentions in the link I provided and the quote from that link. Someone measuring the particle might call that a change in rest mass.

2) With respect to relativity alone the decision to say that the potential energy is subtracted from the external field rather than a property of the particles in a field is a pedagogic one. Further, it is decision which modern particle physicists disagree with probably because in modern particle physics everything is made of fields and the distinction between energy being a property of the excitation of one field vs a property of another non-excited field the excited field is interacting with is unimportant. Seeing as modern particle physics requires one to use both relativity and quantum mechanics and the work towards a total unification of relativity and quantum mechanics is happening in modern particle physics I trust an expert in modern particle physics to have a more accurate model of reality than someone that is just an expert in relativity. Basically I prefer Matt Strasser's pedagogy over Wolfgang Rindler's pedagogy because Matt Strasser should have a more complete and accurate model of reality due to his area of expertise.
Title: Re: If Energy is neither created nor used up, where did energy come from?
Post by: IAMREALITY on 18/06/2016 22:10:40
Where did energy come from=no one knows?
Obviously

When did Energy come from = Energy is eternal?
Our energy came from the big bang

What is Energy = no one knows?
No one knows for certain, but there's a big thread on here detailing what many bright minds believe it to be. 

Was there always energy=Yes?
Ours came from the big bang.  Was there energy to be found external of our Universe, or in multiverses, or whatever, prior to our big bang? Obviously no one knows that either. 

Will there always be energy=Yes?
Probably

Will there be an end to energy=no?
Probably not

Was energy created=No?
Our energy came from the big bang. You're getting repetitive.

Can you touch energy=No
Not a specific enough question, and you'll get a multitude of answers, since there is no definitive definition on what energy is to begin with. 

How much energy exists =Infinity?
No.

How do we find the source of energy=Infinite regression?
No idea what you're even talking about here.  Source of energy? There's not like some energy creating pump out there. 

How long has there been energy = Eternity
In our Universe, since the big bang. 

Are Energy and God the same thing=Yes?
God lmao.  No, there's no magic man in the sky, that's just silliness.  But even if, I fail to see how the two are even remotely related enough in concept to be equated within a logical argument. 

Does Energy contain all information=Yes?
Ummm... No.
Title: Re: If Energy is neither created nor used up, where did energy come from?
Post by: IAMREALITY on 18/06/2016 23:51:18
You are making the assumption that the Big Bang was both the beginning and source of everything, however, logic tells me that there must have been a form of existence before the event of the big bang and this is what I meant by stating "Infinite Regression", remember the Big Bang event is just a theory, albeit the best one, of what happened at the very start of our universe. If there were a something before that it must be just be an assumption and that what is what I was doing in my last post on this subject, namely guessing or speculating as to what, where and how, etc, about the primordial source of energy, indeed if such a thing exists as a reality within existence before and beyond the confines of our universe?

I was making no assumption, but rather answering what was asked. Now if your question is what existed prior to the big bang, well that's a different question altogether, and more a philosophical one and exercise in futility more than anything else since no one knows.  But there are a million hypothetical discussions about what may or may not have caused the big bang or what may have existed before it, online for your perusal. You're free to explore. Www.google.com

As for my thoughts? I have several, depending on my mindset.  But generally, believe that either it all started with the big bang for reasons unknown, that this source of indescribable energy become unstable and burst into existence, and that the energy itself could be literally the energy of nothingness, but the concept still boggles my mind.  Or I may believe in the brane theory, that two colliding branes caused the rupture that on our end became the big bang, or I may believe it could be the product of some other universes black hole, or also what simply happens in the multiverse, that there's this soup of extro-universal state energy that our physics simply can't describe, and that its becoming unstable and bursting universes into existence is simply what it does.

Course, at times I might find myself tripping my friggin face off, I might instead believe that a separate universe had too much dark energy in a runaway expansion, which in time would cause that universe to tear, so an advanced race within it came up with a device that could suck in all the energy and direct it into a tight beam, and focused on a single sub atomic point of spacetime, and that once enough energy was siphoned and beamed into this point, it would cause a new external universe to be created, our big bang, thereby relieving that universe of the extra energy and giving it to ours, causing their universe to achieve a balance to where the risk of a spacetime tear was abated, and that all the energy within our Universe is from their excess that was given to us. And they repeat as necessary. Cause they figured out that any time a certain threshold of energy is crossed within a singularity type point in spacetime, that spacetime will rupture and the product will be the creation of a new spacetime containing the energy that caused the rupture, and that any time a new space time is created inflation will occur. This, like I said, is a theory reserved for when I'm tripping my face off, however.

But the awesome part about "what happened before or may have existed prior to the big bang?" discussions, is that even the wildest of wild theories can have credibility, because no one has a clue what the real answer to the question is. 

What I do know within me with as much certainty as an opinion can muster, is that no higher level consciousness, no theoretical "god", was necessary, nor will ever be found to have played a role. In the end, it is my strong belief, that when the answers are in fact found, that God will be relegated to nothing more than a field, a force that can create from nothingness.  Not because it bears any consciousness or can hear any prayers or any other gobblygook, but instead merely because it's what it does, and can do. 
Title: Re: If Energy is neither created nor used up, where did energy come from?
Post by: PmbPhy on 19/06/2016 04:31:13
agyejy - I’m going to go over reply #107 again because I recently realized precisely the nature of your mistakes. First of all everything that you’ve posted to date demonstrates that you have quite poor reasoning skills. That’s been clear since the point where confused the action of a field on an object with the objects potential energy of position. You’ve confused yourself with this stuff about “where” the field energy is when in fact there’s no physical meaning to locating energy as being at a place in space. We only use energy density and assign it a location to make things easier to think about and calculate.

You’ve also been completely unable to grasp the most simplest of all facts by which I mean that you couldn’t understand the example I gave to you, i.e. an electron placed in a uniform electric field. There is no physical difference between the electrons being at one position than at another. It’d be impossible to tell the difference. Therefore the mass of the electron simply cannot be a function of position such a field. However the potential energy of the electron varies as one moves along a field line. And that's what it means to say that the mass does not depend on the potential energy of position, i.e. the mass may change when placed in a uniform field but it won't change when moved to a higher potential. This is the part which you're unable to grasp.

You were unable to prove that was wrong so you changed to a straw argument, i.e. an electron in a strong magnetic field. First of all the potential energy of an electron in a magnetic field is zero so the fact that the potential energy not being a function of position is irrelevant. In the second place nobody said that the mass of an object won’t change when it’s placed in a field. That’s the straw here, i.e. you changed the subject from potential energy to field strength. They’re different concepts so your example is bogus. There’s absolutely no reason whatsoever that, in general, the proper mass of a classical object will remain unaffected when placed in either an electric or magnetic field. So once again you’re confusing the issue. This time you confused the notion that potential energy of position is not part of the energy in E = mc2.

Then you made some ignorant remarks about Rindler suggesting that just because he’s an authority in relativity it implies that he doesn’t know quantum mechanics, elementary particle physics or quantum field theory. In fact most people take those courses to earn a PhD in physics. One doesn’t get to be an authority such as Rindler by knowing only one field of physics. It’s a horrible mistake to assume that Rindler doesn’t know Quantum Field Theory, Quantum Electrodynamics or Quantum Gravity. Without that knowledge he wouldn’t know much about Hawking Radiation, etc.
Please stop changing the subject of conversion so that you’ll appear to be correct. I’m trying to help the folks here learn something, in this case what kinds of mass/energy are part of E = mc2.

 First of all your absolute worst mistake is using a straw argument, i.e. you’ve changed the subject from classical mechanics to quantum mechanics. You’ve incorrectly assumed that the subject was about elementary particles when in fact classical mechanics doesn’t address such entities. Relativity is the subject and that’s part of classical mechanics, not quantum mechanics. As such it can address such examples as radiation being absorbed or emitted by a macroscopic body just like Einstein used in his paper on E = mc2.

It’s such a shame when people such as yourself get so confused and try to pass it off as someone else’s mistakes like you did here.
Title: Re: If Energy is neither created nor used up, where did energy come from?
Post by: PmbPhy on 19/06/2016 04:40:34
Quote from: Alan McDougall
Where did energy come from=no one knows?
When did Energy come from = Energy is eternal?
What is Energy = no one knows?
Was there always energy=Yes?
Will there always be energy=Yes?
Will there be an end to energy=no?
Was energy created=No?
Can you touch energy=No
How much energy exists =Infinity?
How do we find the source of energy=Infinite regression?
How long has there been energy = Eternity
Are Energy and God the same thing=Yes?
Does Energy contain all information=Yes?
Hi Alan,

I provided explanations for you regarding these questions. You appear to be saying that all of them are wrong. I'd like to request your reasons for your answers given my explanations. Thanks.
Title: Re: If Energy is neither created nor used up, where did energy come from?
Post by: agyejy on 19/06/2016 07:38:13
agyejy - I’m going to go over reply #107 again because I recently realized precisely the nature of your mistakes.

Kind of unfair of you to do this if I am on your ignore list.

Quote
First of all everything that you’ve posted to date demonstrates that you have quite poor reasoning skills.

Ad hominem.

Quote
That’s been clear since the point where confused the action of a field on an object with the objects potential energy of position.

Never happened.

Quote
You’ve confused yourself with this stuff about “where” the field energy is when in fact there’s no physical meaning to locating energy as being at a place in space. We only use energy density and assign it a location to make things easier to think about and calculate.

I guess Wolfgang Rindler is confused as well then. In case anyone missed it the first time:

Quote from: Rindler
In classical mechanics, a particle moving in an electromagnetic (or gravitational)
field is often said to possess potential energy, so that the sum of its kinetic and
potential energies remains constant. This is a useful ‘book-keeping’ device, but energy
conservation can also be satisfied by debiting the field with an energy loss equal to the
kinetic energy gained by the particle.

So is Rindler confused when he says classical mechanics says that the particle posses potential energy and that in relativity we should consider the field as having lost energy? Clearly something cannot lose energy that doesn't have energy.

Quote
You’ve also been completely unable to grasp the most simplest of all facts by which I mean that you couldn’t understand the example I gave to you, i.e. an electron placed in a uniform electric field. There is no physical difference between the electrons being at one position than at another. It’d be impossible to tell the difference. Therefore the mass of the electron simply cannot be a function of position such a field. However the potential energy of the electron varies as one moves along a field line. And that's what it means to say that the mass does not depend on the potential energy of position, i.e. the mass may change when placed in a uniform field but it won't change when moved to a higher potential. This is the part which you're unable to grasp.

See the funny thing about a conservative force field (and I've said this many times) is that in a conservative force field the sum of kinetic and potential energy doesn't change. Therefore when an electron moves along a field line under the influence of the field it loses potential energy and gains an exactly equal amount of kinetic energy. Since the total energy is unchanged the clearly the mass associated with it should not change. It is also important to note that you just admitted that placing an electron in a field may change its mass. Where was the electron before it was in the field? Clearly it was not in the field and thus at a different potential. Ergo a change in potential caused a change in mass. Now if you let that charge move freely under the influence of the field the energy does not change because as I said before potential energy is converted into an equal amount of kinetic energy and total energy doesn't change because the field you describe is conservative. If you force the electron to move from one part of the field to another part of the field (say by picking it up and moving it) then you have to account for that extra force you applied which does work and changes the energy.

Quote
You were unable to prove that was wrong so you changed to a straw argument, i.e. an electron in a strong magnetic field.

Not true.

Quote
First of all the potential energy of an electron in a magnetic field is zero so the fact that the potential energy not being a function of position is irrelevant.

As I said before it is well known the electrons have a magnetic moment and therefore even when stationary have a nonzero potential energy in a magnetic field. Further, a moving electron definitely has a potential energy in a magnetic field because there is a force acting on it. The force is applied in a direction that cannot do work so the electron stays on a line of equipotential but it still has potential energy. I did make this point very clear in another post but I guess you do have me on ignore. It is too bad really because this statement was a rather big error and it is going to be rather embarrassing for you if you repeat it to someone else that knows electromagnetism.

Quote
In the second place nobody said that the mass of an object won’t change when it’s placed in a field.

Clearly if the mass of an object changes when it is placed in a field the amount it changes by should be affected by the properties of the field. Certainly the magnitude of the field which controls the potential energy should have something to do with it. If we force a particle to move around a field to different potentials (i.e. not letting it move from place to place under the influence of the field alone) then clearly the total energy changes and thus the mass. This is analogous to taking a particle from outside the field and forcing it into the field. The reason the mass does not depend on the position of the particle in a conservative force field when that particle moves only under the influence of the field is simply because when undergoing such movement in a conservative field the total energy cannot change. This has nothing to do with the potential energy not counting toward the mass.

Quote
That’s the straw here, i.e. you changed the subject from potential energy to field strength. They’re different concepts so your example is bogus. There’s absolutely no reason whatsoever that, in general, the proper mass of a classical object will remain unaffected when placed in either an electric or magnetic field. So once again you’re confusing the issue. This time you confused the notion that potential energy of position is not part of the energy in E = mc2.

Potential energy is governed explicitly by field strength. In order to place an object in a field it must have first been outside of the field. If the object was outside of the field then it had a different potential energy after you put it into the field. Thus a change in potential energy related with position (outside the field and then inside the field) has caused a change in mass. At this point I should note this is different than the changes due to mass renormalization and only occurs if you move the particle in a way that does not conserve total energy. For example, an electron and a proton forming a Hydrogen atom after the electron emits a photon and thus changes its energy.

Quote
Then you made some ignorant remarks about Rindler suggesting that just because he’s an authority in relativity it implies that he doesn’t know quantum mechanics, elementary particle physics or quantum field theory. In fact most people take those courses to earn a PhD in physics. One doesn’t get to be an authority such as Rindler by knowing only one field of physics. It’s a horrible mistake to assume that Rindler doesn’t know Quantum Field Theory, Quantum Electrodynamics or Quantum Gravity. Without that knowledge he wouldn’t know much about Hawking Radiation, etc.

To be clear I didn't say he doesn't know any of those things. What I said was he is less of an expert than say Matt Strassler. I said that because Rindler has never published any peer reviewed articles on particle physics, quantum mechanics, or quantum field theory. His faculty website lists his expertise as "Basic problems in General Relativity, Relativistic Theoretical Cosmology". Meanwhile Matt Strassler's faculty page lists his expertise as "Particle Physics, Quantum Field Theory, String Theory". Now which one of those two people do you think knows more about quantum field theories? Which one of them should be considered an expert on quantum field theory?

It is also important to point out that a degree (even a PhD) in one field of physics does not make you an expert in every field of physics. This is one of the many reasons I don't rely on my educational qualifications to support my arguments. I support my arguments with citations from people that are clearly experts and that themselve provide evidence for their statements. This is how one should present an argument.

Quote
Please stop changing the subject of conversion so that you’ll appear to be correct. I’m trying to help the folks here learn something, in this case what kinds of mass/energy are part of E = mc2.

Haven't changed the subject at all.

Quote
First of all your absolute worst mistake is using a straw argument, i.e. you’ve changed the subject from classical mechanics to quantum mechanics. You’ve incorrectly assumed that the subject was about elementary particles when in fact classical mechanics doesn’t address such entities. Relativity is the subject and that’s part of classical mechanics, not quantum mechanics. As such it can address such examples as radiation being absorbed or emitted by a macroscopic body just like Einstein used in his paper on E = mc2.


The subject was the nature of energy. Classical mechanics does not have sole dominion over the concept of energy. Furthermore, it is well known that classical mechanics, relativity, and quantum mechanics are all approximations. In fact if you polled physicists I bet most are of the opinion that quantum mechanics is the most fundamental and therefore the closest to reality of the three. This is why the majority of attempts to fully unify relativity and quantum mechanics are focused on quantizing gravity not on doing what would be the reverse on quantum mechanics (I'm not sure if it even has a name).

Quote
It’s such a shame when people such as yourself get so confused and try to pass it off as someone else’s mistakes like you did here.

That's quite clearly not what is happening.

Now since I guess you've got me on ignore you won't be reading this reply. Personally I feel it is quite rude to attempt to refute a person's arguments while doing the digital equivalent of putting your fingers in your ears and yelling "La La I can't hear you!". If you wish to ignore me than do so. It is perfectly within your rights to do so and honestly I can live without the insults. However, I ask that you cease responding to any of my posts because it is very rude to talk to/about someone and then refuse to hear that person's response. My only hope is that either I'm not actually on ignore yet or that at least this part of the message will somehow find its way to you.
Title: Re: If Energy is neither created nor used up, where did energy come from?
Post by: IAMREALITY on 20/06/2016 00:09:29

"If we go further than even the Big Bang into the very depths of the fundamentals, we come across Brane and String Theories that might ultimately give the answer as to the origin of all the energy in our universe (My comment Alan)"

--------------------------------------------------------------------------------------

Reference https://www.physicsforums.com/threads/how-where-did-the-point-of-energy-come-into-existance-frm-for-the-big-bang-to-occour.206466/

Well It seems that String Theory(ST) can provide an answer. ST predicts that the smallest composition of matter are tiny vibrating strands of energy called strings. The way these string wiggle and depending upon whether they are closed or open ended they make up the fundamental particle that we see.

The messenger particle responsible for the transmission of gravity is the gravition. Its is presumed that all the open ended strings are tied to a 3-D surfaces called Branes that floats in a higher dimensional space. There can be many branes that carry parallel universes.

It is believed that two Branes, floating parallely may collide. The energy produced in that collision must have to go somewhere. Where does it go? It goes into the Big Bang. It creates the rapid expansion that we see. ST also provides an answer to why gravity is much weaker than all other fundamental forces of the universe.

The Strings that make up the graviton are open ended. So they may not be confined to the brane that contains our universe. They can seep off in a higher dimensional space. However String Theory is presently untested. The Strings are too small to be dected by the present day accelerators. The Large Hadron Collider which is much powerful than all previous accelerators will primarily search for Something called 'sparticles'- the superpartner of every fundamental particle.

Yes... I touched upon this earlier when I gave a few theories on where the universe originated
Title: Re: If Energy is neither created nor used up, where did energy come from?
Post by: IAMREALITY on 20/06/2016 22:18:52
Quote from: Alan McDougall
Where did energy come from=no one knows?
When did Energy come from = Energy is eternal?
What is Energy = no one knows?
Was there always energy=Yes?
Will there always be energy=Yes?
Will there be an end to energy=no?
Was energy created=No?
Can you touch energy=No
How much energy exists =Infinity?
How do we find the source of energy=Infinite regression?
How long has there been energy = Eternity
Are Energy and God the same thing=Yes?
Does Energy contain all information=Yes?
Hi Alan,

I provided explanations for you regarding these questions. You appear to be saying that all of them are wrong. I'd like to request your reasons for your answers given my explanations. Thanks.


Hi Alan,

I provided explanations for you regarding these questions. You appear to be saying that all of them are wrong. I'd like to request your reasons for your answers given my explanations. Thanks.


Hi Pete I looked it up that was IAMREALITY answering my questions and not me responding to you?

Quote
Where did energy come from="no one knows?" (IAMREALITY SAID THAT)
When did Energy come from = "Energy is eternal?" (IAMREALITY SAID THAT)
What is Energy = "no one knows?"  (IAMREALITY SAID THAT)
Was there always energy="Yes?"  (IAMREALITY SAID THAT)
Will there always be energy="Yes?"  (IAMREALITY SAID THAT)
Will there be an end to energy="no?"  (IAMREALITY SAID THAT)
Was energy created="No?"  (IAMREALITY SAID THAT)
Can you touch energy="No"  (IAMREALITY SAID THAT)
How much energy exists ="Infinity?"  (IAMREALITY SAID THAT)

Alan

Ummmm, where did I say these things exactly?  I literally have no idea what you're talking about or why you're quoting me as having said these things.
Title: Re: If Energy is neither created nor used up, where did energy come from?
Post by: IAMREALITY on 20/06/2016 22:34:32
Quote from: Alan McDougall
Where did energy come from=no one knows?
When did Energy come from = Energy is eternal?
What is Energy = no one knows?
Was there always energy=Yes?
Will there always be energy=Yes?
Will there be an end to energy=no?
Was energy created=No?
Can you touch energy=No
How much energy exists =Infinity?
How do we find the source of energy=Infinite regression?
How long has there been energy = Eternity
Are Energy and God the same thing=Yes?
Does Energy contain all information=Yes?
Hi Alan,

I provided explanations for you regarding these questions. You appear to be saying that all of them are wrong. I'd like to request your reasons for your answers given my explanations. Thanks.


Hi Alan,

I provided explanations for you regarding these questions. You appear to be saying that all of them are wrong. I'd like to request your reasons for your answers given my explanations. Thanks.


Hi Pete I looked it up that was IAMREALITY answering my questions and not me responding to you?

Quote
Where did energy come from="no one knows?" (IAMREALITY SAID THAT)
When did Energy come from = "Energy is eternal?" (IAMREALITY SAID THAT)
What is Energy = "no one knows?"  (IAMREALITY SAID THAT)
Was there always energy="Yes?"  (IAMREALITY SAID THAT)
Will there always be energy="Yes?"  (IAMREALITY SAID THAT)
Will there be an end to energy="no?"  (IAMREALITY SAID THAT)
Was energy created="No?"  (IAMREALITY SAID THAT)
Can you touch energy="No"  (IAMREALITY SAID THAT)
How much energy exists ="Infinity?"  (IAMREALITY SAID THAT)

Alan

This is the history of the post at hand:



Where did energy come from=no one knows?
When did Energy come from = Energy is eternal?
What is Energy = no one knows?
Was there always energy=Yes?
Will there always be energy=Yes?
Will there be an end to energy=no?
Was energy created=No?
Can you touch energy=No
How much energy exists =Infinity?
How do we find the source of energy=Infinite regression?
How long has there been energy = Eternity
Are Energy and God the same thing=Yes?
Does Energy contain all information=Yes?
Where did energy come from=no one knows?
Obviously

When did Energy come from = Energy is eternal?
Our energy came from the big bang

What is Energy = no one knows?
No one knows for certain, but there's a big thread on here detailing what many bright minds believe it to be. 

Was there always energy=Yes?
Ours came from the big bang.  Was there energy to be found external of our Universe, or in multiverses, or whatever, prior to our big bang? Obviously no one knows that either. 

Will there always be energy=Yes?
Probably

Will there be an end to energy=no?
Probably not

Was energy created=No?
Our energy came from the big bang. You're getting repetitive.

Can you touch energy=No
Not a specific enough question, and you'll get a multitude of answers, since there is no definitive definition on what energy is to begin with. 

How much energy exists =Infinity?
No.

How do we find the source of energy=Infinite regression?
No idea what you're even talking about here.  Source of energy? There's not like some energy creating pump out there. 

How long has there been energy = Eternity
In our Universe, since the big bang. 

Are Energy and God the same thing=Yes?
God lmao.  No, there's no magic man in the sky, that's just silliness.  But even if, I fail to see how the two are even remotely related enough in concept to be equated within a logical argument. 

Does Energy contain all information=Yes?
Ummm... No.

You are making the assumption that the Big Bang was both the beginning and source of everything, however, logic tells me that there must have been a form of existence before the event of the big bang and this is what I meant by stating "Infinite Regression", remember the Big Bang event is just a theory, albeit the best one, of what happened at the very start of our universe. If there were a something before that it must be just be an assumption and that what is what I was doing in my last post on this subject, namely guessing or speculating as to what, where and how, etc, about the primordial source of energy, indeed if such a thing exists as a reality within existence before and beyond the confines of our universe?


I was making no assumption, but rather answering what was asked. Now if your question is what existed prior to the big bang, well that's a different question altogether, and more a philosophical one and exercise in futility more than anything else since no one knows.  But there are a million hypothetical discussions about what may or may not have caused the big bang or what may have existed before it, online for your perusal. You're free to explore. Www.google.com

As for my thoughts? I have several, depending on my mindset.  But generally, believe that either it all started with the big bang for reasons unknown, that this source of indescribable energy become unstable and burst into existence, and that the energy itself could be literally the energy of nothingness, but the concept still boggles my mind.  Or I may believe in the brane theory, that two colliding branes caused the rupture that on our end became the big bang, or I may believe it could be the product of some other universes black hole, or also what simply happens in the multiverse, that there's this soup of extro-universal state energy that our physics simply can't describe, and that its becoming unstable and bursting universes into existence is simply what it does.

Course, at times I might find myself tripping my friggin face off, I might instead believe that a separate universe had too much dark energy in a runaway expansion, which in time would cause that universe to tear, so an advanced race within it came up with a device that could suck in all the energy and direct it into a tight beam, and focused on a single sub atomic point of spacetime, and that once enough energy was siphoned and beamed into this point, it would cause a new external universe to be created, our big bang, thereby relieving that universe of the extra energy and giving it to ours, causing their universe to achieve a balance to where the risk of a spacetime tear was abated, and that all the energy within our Universe is from their excess that was given to us. And they repeat as necessary. Cause they figured out that any time a certain threshold of energy is crossed within a singularity type point in spacetime, that spacetime will rupture and the product will be the creation of a new spacetime containing the energy that caused the rupture, and that any time a new space time is created inflation will occur. This, like I said, is a theory reserved for when I'm tripping my face off, however.

But the awesome part about "what happened before or may have existed prior to the big bang?" discussions, is that even the wildest of wild theories can have credibility, because no one has a clue what the real answer to the question is. 

What I do know within me with as much certainty as an opinion can muster, is that no higher level consciousness, no theoretical "god", was necessary, nor will ever be found to have played a role. In the end, it is my strong belief, that when the answers are in fact found, that God will be relegated to nothing more than a field, a force that can create from nothingness.  Not because it bears any consciousness or can hear any prayers or any other gobblygook, but instead merely because it's what it does, and can do.

Yo dude, you asked the questions.  I gave my answers to them randomly.  But nowhere did I say the things you're attributing to me. Above is the short history around the post.  I would ask that you not call me out by name but if you must, please don't attribute quotes to me that are false.  Thank you.
Title: Re: If Energy is neither created nor used up, where did energy come from?
Post by: PmbPhy on 21/06/2016 13:11:56
Quote from: timey
Pete - I do not understand the physicality of 1/square root 1 etc, and what this term relates to within the physical process.  m is mass, v is velocity, e is energy of various types, squaring is a mathematical process, where does the 1 originate from?  What is 1?
I don't think that it's possible to look at all equations in physics and be able to ask "What is the physical meaning of this exponent." or somethin

However in this case the presence of gamma factor in that expression can be traced to the derivation of the expression for time dilation. See: http://www.newenglandphysics.org/physics_world/sr/time_dilation.htm

In that derivation you'll noticed that the Pythagorean Theorem was used. As far as for the physical meaning of each of the terms in that theorem please follow the derivation. All you need to know is basic algebra and the Pythagorean Theorem. The resulting equation is:

(ct/2)2 = (vt/2)2  + (cT/2)2

When you solve for t you get

t = T/sqrt{1 - (v/c)2}

Kinetic energy is then found to be K = (gamma - 1)mc2[/sub].

Quote from: timey
Are you saying that the mathematical process of calculating KE changes above a certain velocity?
You cn
Not precisely. The definition of kinetic energy is defined independent of any theory. You can look it up under the Wok-Kinetic Energy Theorem. If you obtain the expression for kinetic energy in relativity then it becomes the value for classical mechanics as v/c << 1. You don't appear how this can happen. The rule that allows this to happen for such expressions is called the Correspondence Principle.

Consider how this works with time dilation. Recall the expression for it: t = T/sqrt{1 - (v/c)2}

In non-relativistic physics we have v << c (this is to be read "V is much less than the speed of light, c). When this is true the value v/c is negligible compared to one. When this is true the square of v/c is even smaller!!! So as a very good approximation we can set gamma  ~ 1 and therefore t = T and thus there is no time dilation. A similar thing happens with the expression for kinetic energy. Would you like me to work that out for you as well or would                         
Title: Re: If Energy is neither created nor used up, where did energy come from?
Post by: JohnDuffield on 21/06/2016 14:44:21
...Well It seems that String Theory(ST) can provide an answer. ST predicts that the smallest composition of matter are tiny vibrating strands of energy called strings. The way these string wiggle and depending upon whether they are closed or open ended they make up the fundamental particle that we see.

The messenger particle responsible for the transmission of gravity is the gravition. Its is presumed that all the open ended strings are tied to a 3-D surfaces called Branes that floats in a higher dimensional space. There can be many branes that carry parallel universes.

It is believed that two Branes, floating parallely may collide. The energy produced in that collision must have to go somewhere. Where does it go? It goes into the Big Bang. It creates the rapid expansion that we see. ST also provides an answer to why gravity is much weaker than all other fundamental forces of the universe.
I'm afraid this is all speculation Alan. String theory predicts nothing. There is no evidence whatsoever for branes or higher dimensional space.   
Title: Re: If Energy is neither created nor used up, where did energy come from?
Post by: jeffreyH on 21/06/2016 14:53:15
...Well It seems that String Theory(ST) can provide an answer. ST predicts that the smallest composition of matter are tiny vibrating strands of energy called strings. The way these string wiggle and depending upon whether they are closed or open ended they make up the fundamental particle that we see.

The messenger particle responsible for the transmission of gravity is the gravition. Its is presumed that all the open ended strings are tied to a 3-D surfaces called Branes that floats in a higher dimensional space. There can be many branes that carry parallel universes.

It is believed that two Branes, floating parallely may collide. The energy produced in that collision must have to go somewhere. Where does it go? It goes into the Big Bang. It creates the rapid expansion that we see. ST also provides an answer to why gravity is much weaker than all other fundamental forces of the universe.
I'm afraid this is all speculation Alan. String theory predicts nothing. There is no evidence whatsoever for branes or higher dimensional space.   

I have to agree with John on this one. Don't pass out John. String theory operates at a scale where tests of its hypothesis are not possible. If it were to start giving results applicable to larger measurable scales that would be different.
Title: Re: If Energy is neither created nor used up, where did energy come from?
Post by: IAMREALITY on 21/06/2016 16:20:22
...Well It seems that String Theory(ST) can provide an answer. ST predicts that the smallest composition of matter are tiny vibrating strands of energy called strings. The way these string wiggle and depending upon whether they are closed or open ended they make up the fundamental particle that we see.

The messenger particle responsible for the transmission of gravity is the gravition. Its is presumed that all the open ended strings are tied to a 3-D surfaces called Branes that floats in a higher dimensional space. There can be many branes that carry parallel universes.

It is believed that two Branes, floating parallely may collide. The energy produced in that collision must have to go somewhere. Where does it go? It goes into the Big Bang. It creates the rapid expansion that we see. ST also provides an answer to why gravity is much weaker than all other fundamental forces of the universe.
I'm afraid this is all speculation Alan. String theory predicts nothing. There is no evidence whatsoever for branes or higher dimensional space.   

I have to agree with John on this one. Don't pass out John. String theory operates at a scale where tests of its hypothesis are not possible. If it were to start giving results applicable to larger measurable scales that would be different.

I'm not sure he was trying to say it as any sort of accepted science, but rather just informing what ST has to 'say' on the matter.  I'm confused how it can be said ST doesn't predict something because there's no evidence that branes exist or that there's higher dimensional space.  That's the whole point of something being a prediction; there's not yet any evidence of it.  I don't flip a coin, see it land on heads, and then say "I predict it will be heads".  And even if I didn't know coins exist at all, I could still say if there was a coin, and it had two sides, and I flipped it, I predict it will land on heads".  I don't see how someone could then say I predicted nothing.

Now granted, there has been a lot that has poked holes in ST in the last decade or whatnot, and most physicists no longer take it seriously at all (though most also never did).  But it's still a theory that exists that has details to it, and Alan was merely repeating those details.  I don't think he actually believes in ST, I think he was just trying to show us that he's heard of it and has knowledge of it, and posted it merely as a discussion point (I think it was more just a "hey look!  I know stuff!" kinda exercise than a "This is what I believe" one...).  I had alluded to it as well, as one of the many things out there that try and explain pre-Big Bang events.  But I don't believe in ST at all.  But there's nothing wrong with talking about what it has to say on the matter, even though I agree with both of your critiques that there's not a whole lot of meat on the bone insofar as evidence or legitimacy are concerned in respect towards it.